ورود به حساب

نام کاربری گذرواژه

گذرواژه را فراموش کردید؟ کلیک کنید

حساب کاربری ندارید؟ ساخت حساب

ساخت حساب کاربری

نام نام کاربری ایمیل شماره موبایل گذرواژه

برای ارتباط با ما می توانید از طریق شماره موبایل زیر از طریق تماس و پیامک با ما در ارتباط باشید


09117307688
09117179751

در صورت عدم پاسخ گویی از طریق پیامک با پشتیبان در ارتباط باشید

دسترسی نامحدود

برای کاربرانی که ثبت نام کرده اند

ضمانت بازگشت وجه

درصورت عدم همخوانی توضیحات با کتاب

پشتیبانی

از ساعت 7 صبح تا 10 شب

دانلود کتاب Surgery. A Case Based Clinical Review

دانلود کتاب عمل جراحی. بررسی بالینی مبتنی بر مورد

Surgery. A Case Based Clinical Review

مشخصات کتاب

Surgery. A Case Based Clinical Review

ویرایش: 2 
نویسندگان:   
سری:  
ISBN (شابک) : 9783030053864, 9783030053871 
ناشر: Springer 
سال نشر: 2020 
تعداد صفحات: 652 
زبان: English 
فرمت فایل : PDF (درصورت درخواست کاربر به PDF، EPUB یا AZW3 تبدیل می شود) 
حجم فایل: 12 مگابایت 

قیمت کتاب (تومان) : 34,000



ثبت امتیاز به این کتاب

میانگین امتیاز به این کتاب :
       تعداد امتیاز دهندگان : 17


در صورت تبدیل فایل کتاب Surgery. A Case Based Clinical Review به فرمت های PDF، EPUB، AZW3، MOBI و یا DJVU می توانید به پشتیبان اطلاع دهید تا فایل مورد نظر را تبدیل نمایند.

توجه داشته باشید کتاب عمل جراحی. بررسی بالینی مبتنی بر مورد نسخه زبان اصلی می باشد و کتاب ترجمه شده به فارسی نمی باشد. وبسایت اینترنشنال لایبرری ارائه دهنده کتاب های زبان اصلی می باشد و هیچ گونه کتاب ترجمه شده یا نوشته شده به فارسی را ارائه نمی دهد.


توضیحاتی درمورد کتاب به خارجی



فهرست مطالب

Foreword
Preface
Acknowledgments
Contents
Contributors
I: Acute Care Surgery
	1: Fever and Hypotension in the Intensive Care Unit
		Diagnosis
			What Is the Differential Diagnosis for a Post-op Patient with Fever?
			What Is the Classic Timing for the Different Causes of Postoperative Fever (the Five Ws)?
			What Is the Most Likely Diagnosis in This Patient?
			What Other Diagnoses Should Be Considered in a Postoperative Patient with Fever and Hypotension?
		History and Physical
			What Clues Are Expected on History and Physical for a Patient in Sepsis?
			What Is the Sequential Organ Failure Assessment (SOFA) and Quick SOFA (qSOFA) Score?
			What Physical Exam Findings Suggest that a Patient Is Unable to Protect Their Airway?
		Pathophysiology
			What Are the Different Types of Shock?
			What Is the Pathophysiology of Septic Shock?
			What Is Acute Respiratory Distress Syndrome (ARDS)?
			If the Patient Were to Become Pulseless, What Are the Hs and Ts to Consider?
		Workup
			What Are the Initial Laboratory Studies to Order for a Patient in Septic Shock?
			What Is the Significance of Increased Serum Creatinine and Oliguria?
			What Is the Difference Between the Two Types of Renal Replacement Therapy: Hemodialysis (HD) and Continuous Renal Replacement Therapy (CRRT)?
			What Are the Initial Imaging Studies to Order for a Postoperative Patient Believed to Be in Septic Shock?
		Management
			What Are the Initial Management Goals for a Patient in Septic Shock?
			What Is Rapid Sequence Intubation (RSI)?
			What Drugs Are Ideal to Use for Sedation in a Critically Ill and Intubated Patient?
			What Drugs Are Ideal to Use for Analgesia in a Critically Ill and Intubated Patient?
			How Should the Patient’s Oliguria Be Managed?
			What Is the Significance of a Low pH?
			How Are Glucose Levels Managed in the Septic Patient?
			If the Patient Became Pulseless, How Should ACLS Be Executed?
			What Is the Difference Between Synchronized and Desynchronized Shock?
		Summary of Essentials
			History and Physical
			Pathophysiology
			Workup
			Management
		Suggested Reading
	2: Nausea, Vomiting, and Left Groin Mass
		Diagnosis
			What Is the Differential Diagnosis for a Groin Mass (Use MINT Mnemonic)?
			What Is the Diagnosis for This Patient?
		History and Physical
			Why Is It Important to Examine for Inguinal Lymphadenopathy?
			Why Is It Important to Ask if a Groin Mass Protrudes with Straining?
		Pathophysiology
			What Is a Hernia?
			What Is the Difference Between a Reducible and an Incarcerated Hernia? Between an Incarcerated and a Strangulated One?
			What Is the Pathophysiology of an Indirect Inguinal Hernia? A Direct Inguinal Hernia?
			What Are the Borders of Hesselbach’s Triangle?
			What Are the Other Types of Hernias?
			Why Are Femoral Hernias More Prone to Incarceration?
			What Is the Significance of a Suspected Hernia Being Below as Opposed to Above the Inguinal Ligament?
			What Is a Richter’s Hernia?
			What Is a Sliding Hernia?
		Workup
			How Do You Diagnose a Hernia in an Adult?
			How Do You Diagnose a Hernia in an Infant?
			How Do You Distinguish Between a Direct and an Indirect Inguinal Hernia Intraoperatively?
			What Is the Role of Imaging Studies in the Diagnosis of Hernia?
		Management
			What Is the Principle Component of the Operative Management of an Indirect Inguinal Hernia in an Adult? How About a Direct Inguinal Hernia?
			What Are the Principles of the Management of an Incarcerated Hernia?
			What Is the Recommendation for Inguinal Hernia Repair in Older Infants/Children?
			What Is the Recommended Management for an Umbilical Hernia in an Infant?
		Complications
			What Nerves Can Be Injured During Hernia Repair? What Is the Mechanism of Injury? What Are the Consequences?
		Areas Where You Can Get in Trouble
			Attempting to Reduce a Strangulated Hernia
			Dissecting and Excising the Distal End of a Large Indirect Hernia Sac
			Chronically Reducible Hernia with SBO
		Areas of Controversy
			Do You Repair or Observe Asymptomatic Inguinal Hernias?
			What Is the Recommended Management for an Inguinal Hernia in a Premature Infant?
			Do You Repair Asymptomatic Ventral/Incisional Hernias? Is Mesh Needed for Repair?
			Do You Repair Inguinal Hernias Laparoscopically or Open?
		Summary of Essentials
			History and Physical
			Differential Diagnosis
			Pathology/Pathophysiology
			Workup
			Management
			Complications
		Suggested Reading
	3: Abdominal Pain, Nausea, and Vomiting
		Diagnosis
			What Is the Differential Diagnosis?
			What Is the Most Likely Diagnosis?
		History and Physical Exam
			What Is the Significance of SBO in the Absence of an Abdominal Scar?
			What Is the Howship-Romberg Sign?
			What Is the Significance of Severe Abdominal Pain and Localized Tenderness in Association with an SBO?
		Pathophysiology
			What Is a Closed Loop Obstruction?
			What Is the Pathophysiology of SBO?
			Are Bowel Sounds Helping in Diagnosing SBO?
			What Are the Most Common Causes of an SBO?
			What Is the Risk of Developing SBO After Different Operations?
			Does Laparoscopic Surgery Have a Lower Risk for SBO Compared to Open Surgery?
			What Are the Mechanisms of Fluid Loss in SBO?
			What Is the Mechanism of Aciduria in Patients with Ongoing Emesis?
		Workup
			What Laboratory Tests Should Be Obtained in the Initial Workup for SBO?
			What Imaging Is Recommended for an SBO?
			How Do You Differentiate Large and Small Bowel on Radiographs?
			What Are the Different Radiologic Findings Associated with SBO?
			How Is a Complete SBO Different from a Partial SBO? Why Is It Important to Distinguish Between the Two?
			How Do You Distinguish SBO from LBO?
			How Do You Distinguish Between Postoperative Ileus and SBO?
		Management
			What Are the Initial Steps in the Management of an SBO?
			What Is the Role of Gastrografin in the Management of SBO?
			Operative Versus Nonoperative Management of SBO
			What Should You Do if You Suspect Nonviable Bowel During Laparotomy for SBO?
			What Is the Management of Early Postoperative SBO?
		Areas Where You Can Get in Trouble
			Nausea and Vomiting Months to Years After Gastric Bypass
			Abdominal Complaints in Opioid Drug Users
		Summary of Essentials
			History and Physical Exam
			Pathophysiology
			Diagnosis
			Management
		Suggested Reading
	4: Right Leg Pain, Swelling, and Erythema for Two Days
		Diagnosis
			What Is the Differential Diagnosis?
			What Is the Most Likely Diagnosis?
		History and Physical
			What Is the Implication of Crepitus?
			What Are the Risk Factors for NSTI?
			What Is the Implication of Bullae? Violaceous Skin?
			What Are the “Hard Signs” of NSTI? What Percent of Patients with NSTI Have Such Hard Signs?
			Why Is It Important to Distinguish Between Cellulitis and NSTI? How Do Laboratory Values Help?
		Pathophysiology
			What Is the Spectrum of NSTI?
			What Are the Typical Organisms Seen in NSTI?
			What Is the Implication of Culturing Clostridium septicum From the Wound?
			What Is the Term for NSTI that Involves the Scrotum and/or Perineum?
		Management
			What Are the Initial Steps in the Management of NSTI?
			How Do You Determine How Much Tissue to Debride?
		What If Extensive Muscle Necrosis Is Found?
			What Are the Intraoperative Findings that Confirm NSTI?
			What Is the Role of a Second-Look Operation?
			What Do You Do If Your Suspicion for NSTI Is High But You Are Not Certain of the Diagnosis?
			Is Imaging Beneficial in the Diagnosis of NSTI?
			What Is the Anticipated Mortality Risk Associated With NSTI?
		Area Where You Can Get in Trouble
			Postsurgical Wound With Dishwater Purulent Fluid
		Area of Controversy
			What Is the Role of Hyperbaric Oxygen?
		Summary of Essentials
			History and Physical Examination
			Diagnosis
			Management
		Suggested Reading
I: Question Set: Acute Care Surgery
	Questions
	Answers
II: Breast and Skin
	5: Abnormal Screening Mammogram
		Diagnosis
			What Is the Differential Diagnosis for an Abnormal Mammogram in the Absence of a Palpable Breast Mass?
			What Is the Most Likely Diagnosis?
		Screening
			What Are the Recommendations for Breast Cancer Screening?
			What Are the Risks of Mammography?
			Why Is Mammography Not Useful in Young Women (<30 Years Old)?
		History and Physical
			What Is the Gail Risk Model?
		Pathophysiology
			What Is One Characteristic of Mammographic Calcifications that Helps Differentiate Between Benign and Malignant Conditions?
			What Are the Primary Features of Ductal Carcinoma In Situ?
			What Are the Primary Features of Lobular Carcinoma In Situ?
			How Does Breast Cancer Metastasize to the Spine?
		Workup
			How Are Mammograms Used in Staging Breast Lesions?
			What Is the Next Step in Management for the Above Patient?
			What Is a Stereotactic Core Needle Biopsy?
			How Does One Interpret the Results of a Stereotactic Core Needle Biopsy?
			What Is the Next Step if the Stereotactic Biopsy Is Discordant?
			What Other Findings on Core Needle Biopsy Require Excisional Biopsy?
			What Constitutes Positive Margins on a Malignant Specimen?
		Management
			What Is the Next Step if the Stereotactic Needle Biopsy Demonstrates DCIS?
			What Is the Next Step if the Stereotactic Biopsy Demonstrates LCIS?
			What Are the Treatment Options for LCIS?
			What if the Stereotactic Biopsy Shows Invasive Ductal Carcinoma?
			Can You Get Lymph Node Metastasis with DCIS?
			Mastitis that Does Not Resolve with Antibiotics
		Areas of Controversy
			Does Screening Mammography Actually Reduce Mortality from Breast Cancer?
			Prophylactic Bilateral Mastectomy
		Summary of Essentials
			Diagnosis
			Screening
			History and Physical
			Etiology/Pathophysiology
			Workup
			Management
		Suggested Reading
	6: New Palpable Mass in the Right Breast
		Diagnosis
			What Benign Conditions Are in the Differential Diagnosis of a Palpable Breast Mass?
			What Malignant Lesions Are in the Differential Diagnosis of a Palpable Breast Mass?
			What Is the Most Likely Diagnosis for This Patient?
		History and Physical
			What Features on Physical Examination Are Suggestive of Breast Cancer?
			What Are the Risk Factors for Breast Cancer?
			What Are the Different Types of Nipple Discharge and What Is the Differential Diagnosis for Each?
		Pathology/Pathophysiology
			What Histologic Features of Fibrocystic Changes Are Associated with Increased Risk for Cancer?
			What Is the Pathophysiology of “Peau d’ Orange”?
			What Is the Pathophysiology of Nipple Retraction?
		Workup
			What Is the Triple Test for a New Breast Mass?
			How Does the Age of the Patient Affect the Workup of a New, Palpable Breast Mass?
			What Imaging Is Recommended for Working Up a New Breast Mass in Women Over 30?
			What Metastatic Workup Is Recommended Following a Diagnosis of Breast Cancer?
			How Is Breast Cancer Staged Clinically?
			What Are the Different Tumor Markers for Breast Cancer and How Are They Utilized?
			What Is a Triple-Negative Breast Cancer?
			What Breast Cancer Subtype Do Patients with BRCA Develop?
			What Are the Unique Features of Invasive Lobular Carcinoma?
		Management
			What Surgical Options Are Available for Patients with Early (Stage I and II) Breast Cancers?
			What Is the Premise Behind SLNB?
			Why Does SLNB Need to Be Performed at the Same Time as a Mastectomy and Not Later?
			Can There Be More than One Sentinel Node?
			What Do You Do if During SLNB No Sentinel Lymph Node Lights Up?
			What Are Contraindications to BCT?
			Can the Nipple and Breast Skin Be Spared During a Simple Mastectomy?
			What Are the Management Options for Clinically Advanced (Stage III) Breast Cancers?
			When Do You Perform Axillary Lymph Node Dissection (ALND)?
			What Are the Boundaries in the Axilla for Breast Dissection?
			To What Structure Are Axillary Lymph Node Levels Referenced?
			What Is the Purpose of Axillary Lymph Node Dissection? Does It Affect Survival?
			What Are the Options for Hormonal Therapy, and What Is the Premise Behind It?
			What Study Must Be Done Prior to Starting Trastuzumab?
			Why Is an Aromatase Inhibitor (AI) Only Effective in Postmenopausal Women?
			Does Everyone with Breast Cancer Require Chemotherapy?
			What Are the Most Common Chemotherapy Regimens?
			What Are the Options for Breast Reconstruction? What Is the Timing?
		Complications
			What Nerves Are at Risk for Damage During ALND?
			What Is the Most Morbid Complication of Lymph Node Dissection?
		Areas You Can Get in Trouble
			Confusing Inflammatory Breast Carcinoma with Cellulitis
			Ignoring a Breast Mass During Pregnancy
			Ignoring a Breast Mass in an Elderly Male
		Areas of Controversy
			Should Mastectomy Be Performed in Patients with Stage IV (Metastatic) Breast Cancer?
			Should BCT Patients Receive Intraoperative Radiation?
		Summary of Essentials
			History and Physical
			Etiology/Pathophysiology
			Diagnosis
			Management
			Complications
			Watch Out
		Suggested Reading
	7: Recently Changed Skin Lesion
		Case Study
		Diagnosis
			What Is the Differential Diagnosis?
			What Is the Most Likely Diagnosis?
		History and Physical
			What Risk Factors for Skin Cancer Are Common to SCC, BCC, and Melanoma?
			What Factors During Childhood/Teen Years Are Associated with an Increased Risk of Skin Cancer?
			What Genetic Conditions Are Associated with an Increased Risk of Skin Cancer?
			What Occupations Are Highly Associated with Skin Cancer?
			What Are Findings on Physical Examination that Differentiate a Benign Nevus from Melanoma?
			What Is the Ugly Duckling Sign?
			Why Is It Important to Inquire About and Examine Areas of Chronic Skin Inflammation?
			On What Areas of the Skin Are Melanomas Most Likely to Occur in Nonwhite Ethnicities?
			What Is the Most Common Site of Melanoma in Men Versus Women?
			What Is the Most Common Site of Digital Melanoma?
			Does the Regular Use of SPF Protection Reduce the Risk of Skin Cancer?
			What Is an “In-Transit” Metastasis? Satellite Lesion?
		Pathophysiology
			What Is a Nevus? What Are Dysplastic Nevi? Are Nevi a Risk for Malignant Transformation? What Is Dysplastic Nevus Syndrome?
			From Where Does Melanoma Arise?
			What Is the Most Common Skin Cancer? Second Most Common? Which Skin Cancer Is Associated with the Greatest Number of Deaths?
			What Is the Most Common Precancerous Skin Lesion?
			What Is Bowen’s Disease?
			What Is the Metastatic Risk of BCC, SCC, and Melanoma?
			What Are the Four Subtypes of Melanoma?
			What Is the Difference Between Clark Classification and Breslow Depth?
		Workup
			What Are the Different Types of Skin Biopsy?
			Once the Diagnosis of Melanoma Is Established, What Additional Studies Should Be Obtained?
			What Are Poor Prognostic Indicators with Melanoma?
		Management
			What Treatment Options Exist for SCC and BCC?
			What Is the Primary Therapy for BCC/SCC Skin Cancers?
			What Type of Surgical Margins Do You Need for BCC vs. SCC?
			Which Type of Basal Cell Carcinoma Has the Worst Prognosis?
			How Is Melanoma Surgically Managed?
			What Is the Purpose of the SLNB, and How Is It Performed?
			What Are the Indications for Lymph Node Dissection?
			How Is Melanoma of the Fingernail Managed?
			What Is the Prognosis for Melanoma?
			What Is the Follow-Up Protocol for Melanoma?
			What Is Mohs Surgery? What Are the Main Indications for Its Use? Is It Appropriate for the Treatment of Melanoma?
		Areas Where You Can Get in Trouble
			Assuming that a Discolored Nail Bed Is a Benign Condition
			Pink Nodule Progressing to a Blue Color in an Immunocompromised Patient
			Relying Solely on the ABCDE Rule for Detecting Melanomas
			Recurrent Infections Around Gluteal Cleft
		Areas of Controversy
			Is There Any Benefit for Surgical Resection for Stage IV (Distant Metastasis) Melanoma?
			Is Adjuvant Therapy Beneficial for Advanced Melanoma?
			Are There Medical Therapies for Metastatic Melanoma Patients?
			Melanoma Recurring Many Years After Initial Presentation
		Summary of Essentials
			History and Physical
			Differential Diagnosis
			Pathology/Pathophysiology
			Workup
			Management
			Watch Out
		Suggested Reading
II: Question Set: Breast 
and Skin
	Questions
	Answers
III: Cardiothoracic
	8: Chest Pain, Diaphoresis, and Nausea
		Diagnosis
			What Is the Differential Diagnosis?
			What Is the Most Likely Diagnosis?
		History and Physical
			What Are the Risk Factors for Myocardial Infarction (MI)?
			What Elements on the History and Physical Exam Make MI Less Likely in a Patient Presenting with Chest Pain?
			What Are the Classic History and Physical Exam Findings Seen in MI?
			What Group of Women Is at Highest Risk for MI? Why?
			What Are the Three Classic Symptoms of Aortic Stenosis?
			What Are the Differentiating Features of Various Cardiac Murmurs?
			Changes Seen in Systolic Murmurs with Various Maneuvers
		Pathophysiology
			What Is Meant by Acute Coronary Syndrome?
			What Is the Difference Between UA, NSTEMI, and STEMI?
			What Are the Series of Events that Take Place During an Acute Myocardial Infarction?
			What Coronary Vessel Is Most Often Affected?
			Are There Different Mechanisms of Myocardial Infarction?
			What Is Suggested by Episodic Chest Pain Unrelated to Exertion in a Young Person?
			What Is the Dreaded Consequence of Aortic Stenosis?
		Workup
			What Are the Initial Diagnostic Steps for Suspected Myocardial Infarction?
			What Is the Role of Measurement of Cardiac Enzyme Levels in the Blood?
			What Is the Best Cardiac Enzyme to Diagnose a Second MI on Top of a Recent MI?
			What Is the Role of ECG?
			What Is the Role of Imaging?
		Management
			What is the Initial Management of STEMI?
			When Should Nitrates Be Avoided?
			What Options for Reperfusion Are Available for STEMI?
			What if PCI Is Unsuccessful?
			Does the Timing of Intervention Matter?
			What Is the Initial Management of NSTEMI?
			What Are the Next Options in the Management of NSTEMI?
			What Is the Most Important Determinant of Long-Term Outcome After STEMI? Is there a role of emergency CABG after a STEMI?
			What Are the Indications for CABG in the Elective Setting?
			What Does the Traditional Coronary Artery Bypass Grafting (CABG) Entail?
			What Is the Best Conduit for CABG?
			Should More than One Coronary Artery Be Bypassed During CABG?
			How Does CABG Compare to Coronary Artery Stenting in Patients with Multivessel Coronary Artery Disease?
			What Medical Interventions Are Associated with Improved CABG Outcomes in the  Perioperative Period?
			What Are the Predictors of Mortality Following CABG?
			How Is Prinzmetal Angina Managed?
		Complications
			What Are the Complications of MI?
		Areas You Can Get in Trouble
			Silent MI
			Misdiagnosing Aortic Dissection as Acute MI
		Area of Controversy
			Is the Radial Artery Useful as a Graft in CABG?
		Summary of Essentials
			History and Physical
			Pathophysiology
			Workup
			Management
		Suggested Reading
	9: Chest and Back Pain
		Diagnosis
			What Is the Differential Diagnosis?
			What Is the Most Likely Diagnosis?
		History and Physical
			What Are the Risk Factors for Aortic Dissection?
			What Is the Significance of Unequal Pulses in the Upper and Lower Extremities in Patients with Aortic Dissection?
			What Is the Significance of an Increased Pulse Pressure?
			What Is the Significance of New Diastolic Murmur?
			What Is the Significance of a History of Intravenous Drug Abuse (IVDA)?
		Pathophysiology
			What Is the Initial Event Leading to an Aortic Dissection?
			Why Are Patients with Marfan’s Syndrome at Increased Risk for Aortic Dissection?
			Are Dissections and Aneurysms the Same Disease?
			How Are Aortic Dissections Classified?
			How Do Aortic Dissections Cause Complications?
			What Life-Threatening Complications Are Specific to Dissections that Involve the Ascending Aorta/Aortic Arch (Stanford Type A)?
			What Major Complications Are Specific to Dissections that Only Involve the Descending Aorta (Stanford Type B)?
		Workup
			What Laboratory Tests Should Be Ordered Immediately?
			What Is the First Imaging Modality Recommended in a Patient Who Presents with Acute Chest Pain?
			What Imaging Is Recommended if the Patient Is Hemodynamically Unstable and an Aortic Dissection Is Highly Suspected?
		Management
			What Is the First Step in Management of an Aortic Dissection?
			What Is the Next Step (After Starting Antihypertensives) Once the Diagnosis of Type A Dissection Is Established?
			What Is the Next Step (After Starting Antihypertensives) Once a Diagnosis of Type B Dissection Is Established?
			When Should Patients with Type B Dissection Undergo Surgical Repair?
			What Are the Surgical Options for a Type B Dissection?
			What Are the Differences in Prognosis and Definitive Management Between Type A and Type B Aortic Dissections?
		Areas Where You Can Get in Trouble
			Giving Beta-Blockers to a Patient with Aortic Dissection Complicated by Cardiac Tamponade or Severe Aortic Regurgitation
			Confusing Type A Aortic Dissection with Acute MI
		Area of Controversy
			Endovascular Repair of a Stable Asymptomatic Type B Aortic Dissection
		Summary of Essentials
			History and Physical Exam
			Watch Out
			Pathophysiology
			Classification
			Diagnosis
			Management
		Suggested Reading
	10: Hemoptysis, Cough, and Weight Loss
		Diagnosis
			What Is the Differential Diagnosis?
			What Is the Most Likely Diagnosis?
		History and Physical
			What Are the Risk Factors for Lung Cancer?
			What Is the Importance of Occupational Exposure?
		Pathophysiology
			Does This Patient Have a Solitary Pulmonary Nodule?
			How Is Lung Cancer Classified? Why Is It Important?
			What Is the Epidemiology of Lung Cancer?
			What Syndromes Are Often Associated with Lung Cancers?
			What Are Common Sites of Metastasis for Primary Lung Cancer?
		Workup
			How Does One Determine the Likelihood of Malignancy When a Pulmonary Lesion Is Detected on Chest X-Ray?
			What Are the Initial Diagnostic Steps for Suspected Lung Cancer?
			What Is the Role of PET Scan?
			Should a Biopsy Be Performed?
		Management
			How Does the Lung Cancer Type Influence Management?
			What Is the Importance of Preoperative Evaluation?
		Areas of Controversy
			Is Universal Screening for Lung Cancer Appropriate?
			Is Screening of Benefit to Any Group?
		Summary of Essentials
			History and Physical Exam
			Watch Out
			Pathophysiology
			Classification
			Diagnosis
			Management
		Suggested Reading
III: Question Set: 
Cardiothoracic
	Questions
	Answers
IV: Endocrine
	11: Incidentally Discovered Adrenal Mass on CT Scan
		Diagnosis
			What Is Meant by the Term Adrenal Incidentaloma?
			What Is the Differential Diagnosis for an Incidentally Discovered Adrenal Nodule?
			What Is the Most Likely Diagnosis?
		History and Physical
			What Are the Findings on History and Physical Examination in a Patient that Hypersecretes Cortisol?
			What Is the Difference Between Cushing’s Syndrome and Cushing’s Disease?
			What Is the Clinical Presentation of an Adrenal Nodule that Hypersecretes Aldosterone?
			What Is the Clinical Presentation of a Pheochromocytoma?
			What Signs and Symptoms Should Raise Suspicion for an Adrenocortical Carcinoma?
		Pathophysiology
			What Is the Most Common Adrenal Mass?
			What Are the Zones of the Adrenal Gland and What Hormones Do They Produce?
			What Are the Physiologic Effects of Normal and Excessive Cortisol Secretion?
			What Is the Cause of Hypertension and Hypokalemia in Hyperaldosteronism?
		Workup
			What Is the Next Step in Workup when an Adrenal Adenoma Is Suspected or Seen on Imaging?
			What Is the Difference Between a Low-Dose and High-Dose Dexamethasone Suppression Test?
			What Laboratory Testing Can Identify Hyperaldosteronism?
			What Is Adrenal Vein Sampling? What Is Its Role in Evaluating Hyperaldosteronism?
			What Laboratory Testing Can Identify a Pheochromocytoma?
			What Is the Best Imaging Modality to Evaluate an Adrenal Nodule? What Is Another Option?
			What Imaging Characteristics Help to Differentiate a Benign from Malignant Lesion?
		Management
			What Is the Treatment for a Nonfunctional Adrenal Mass? How Does the Size Impact Management?
			What Is the Surveillance Protocol for an Adrenal Nodule that Will Not Be Resected?
			What Is the Treatment for a Functional Adrenal Mass?
			What Are Important Perioperative Management Principles?
			What Are the Key Surgical Principles During an Adrenalectomy?
		Prognosis
			What Is the Prognosis of an Adrenal Adenoma?
		Key Areas Where You Can Get in Trouble
			What Is the Role of Percutaneous Biopsy in the Workup of an Adrenal Mass?
			What Is Subclinical Cushing’s Syndrome and How Should It Be Managed?
		Summary of Essentials
			History and Physical
			Etiology/Pathophysiology
			Diagnosis
			Management
			Watch Out
		Suggested Reading
	12: Fatigue, Constipation, and Depressed Mood
		Diagnosis
			What Is the Differential Diagnosis of Hypercalcemia?
			What Is This Patient’s Diagnosis?
		History and Physical
			What Is the Typical Presentation for Patients with Hypercalcemia?
			What Are the Renal Manifestations of Hypercalcemia?
			What Are the Gastrointestinal Manifestations of Hypercalcemia?
			What Are the Neurological Manifestations of Hypercalcemia?
			What Patient Demographic Most Commonly Presents with Hyperparathyroidism?
			What Are the Risk Factors for Primary Hyperparathyroidism?
			Why Is Family History Important?
			What Is a Hypercalcemic Crisis?
			What Are the Physical Exam Findings of Hyperparathyroidism? What Is the Significance of an Anterior Neck Mass Palpated in a Patient with Hyperparathyroidism?
			What Is Chvostek’s Sign?
			What Is Trousseau’s Sign?
			What Is T-Score?
		Anatomy
			Describe the Location of the Parathyroid Glands. What Is Their Embryological Development?
		Pathology/Pathophysiology
			What Pathology Causes Primary Hyperparathyroidism? Is There a Genetic Component?
			What Is the Typical Pathology in Primary Hyperparathyroidism Associated with the MEN Disorders?
			What Is the Pathophysiology of Secondary Hyperparathyroidism?
			What Causes Tertiary Hyperparathyroidism?
			What Are the End Organs Affected by Parathyroid Hormone?
			What Is the Basic Physiology of Calcium Metabolism?
			What Is the Most Common Cause of Parathyroid Hormone Deficiency? How Does It Manifest?
		Work-Up
			What Laboratory Tests Are Used for the Diagnosis of Primary Hyperparathyroidism?
			Does an Elevated PTH Level Combined With an Elevated Serum Calcium Level Establish the Diagnosis of Primary Hyperparathyroidism?
			What Is the Difference Between Total Serum Calcium Level and Ionized Calcium Level?
			If the Serum Calcium Is High, but the PTH Level Is Normal, Does That Rule Out Primary Hyperparathyroidism?
			How Can the Serum Chloride to  Phosphate Ratio Suggest Primary Hyperparathyroidism?
			What Other Tests Should Be Ordered in a New Diagnosis of Hyperparathyroidism?
			What Radiologic Findings Are Suggestive of Bony Involvement in Hyperparathyroidism?
			What Tests Help Localize the Involved Gland in Hyperparathyroidism?
			What If All Localizing Scans Fail to Localize the Abnormal Parathyroid Gland(s)?
		Management
			What Are the Indications for Parathyroidectomy in Primary Hyperparathyroidism?
			What Are the Indications for Parathyroidectomy in Secondary Hyperparathyroidism?
			What Is the Nonoperative Management of Patients with Hyperparathyroidism?
			What Is the Role of Intraoperative PTH Monitoring?
			What Are the Surgical Options for Hyperparathyroidism?
			Why Is It Advised Not to Remove All Four Parathyroid Glands?
			What Is the Postoperative Management Following Parathyroidectomy?
			What Is the Treatment for a Neck Hematoma with Stridor?
			What Are the Benefits of Parathyroidectomy?
			What Syndrome Causing Postoperative Hypocalcemia Can Occur Following Parathyroidectomy?
		Areas of Controversy
			Surgery Versus Observation in Asymptomatic Hyperparathyroidism
			Focused/Unilateral Exploration Versus Four-Gland Exploration, Use of Intraoperative PTH Testing
		Areas Where You Can Get in Trouble
			Assuming Normal Calcium Rules Out Hyperparathyroidism
			Hypocalcemia That Does Not Respond to Calcium Replacement
		Special Situations
			How Do You Treat Hypercalcemic Crisis?
			Algorithm for the Work-Up and Management of Hyperparathyroidism (. Figs. 12.3 and 12.4)
		Summary of Essentials
			History and Physical
			Diagnosis
			Pathophysiology
			Management
			Watch Out
		Suggested Reading
	13: Intermittent Episodes of Sweating, Palpitations, and Hypertension
		Diagnosis
			What Is the Differential Diagnosis and What Clues on History and Physical Examination Might Direct You Toward a Specific Diagnosis?
			What Is the Most Likely Diagnosis?
		History and Physical
			What Is the Differential Diagnosis for Surgically Correctable Hypertension (HTN)?
			What Is the Classic Triad of Symptoms in Pheochromocytoma?
			What Are the Other Symptoms of Pheochromocytoma?
			Why Do Some Patients Have Orthostatic Hypotension?
			Can Patients with Pheochromocytoma Be Asymptomatic?
			What Is the Average Age of Diagnosis? Is There a Gender Predisposition?
			What Is the “Rule of Tens,” and Is It Still True?
			In a Young Patient with Hypertension, Why Is It Important to Check Blood Pressure in Both the Arms and Legs?
		Pathophysiology
			What Is a Pheochromocytoma?
			What Are the Effects of Normal and Excessive Catecholamine Secretion?
			What Are the Predominant Catecholamines in Adrenal and Extra-Adrenal Tumors?
			Where Do Pheochromocytomas Develop?
			What Can Trigger a Hypertensive Crisis in Pheochromocytoma?
			Why Do Patients with Pheochromocytoma Have Elevated Hematocrit Levels?
			Why Do Patients With Pheochromocytoma Have Hyperglycemia?
			What Defines a Malignant Pheochromocytoma?
		Workup
			What Laboratory Tests Can Help Establish the Diagnosis of Pheochromocytoma?
			Is Urine or Plasma Testing More Reliable?
			What Medications Can Interfere with Urine or Plasma Testing?
			What Imaging Studies Are Helpful in Making the Diagnosis?
		Management
			What Is the Treatment for Pheochromocytoma?
			What Are the Important Surgical Principles?
			How Do You Prepare a Patient for Surgery?
			What Can Happen If a Beta-Blocker Is Given Prior to Adequate Alpha-Blockade?
			When Is Genetic Testing Indicated?
			What Are the Main Complications Specifically Associated with Adrenalectomy for Pheochromocytoma?
			How Often Should Plasma and Urinary Catecholamines Be Measured Postoperatively?
		Prognosis
			What Is the Overall Prognosis for Pheochromocytoma?
			What Is the Treatment for Malignant Pheochromocytoma?
		Key Areas Where You Can Get in Trouble
			Do Not Biopsy a Pheochromocytoma
			Pheochromocytoma and Pregnancy
			Hypertensive Crisis During Surgery Due to Undiagnosed Pheochromocytoma
		Summary of Essentials
			History and Physical
			Etiology/Pathophysiology
			Diagnosis
			Management
			Watch Out
		Suggested Reading
	14: Neck Mass that Moves with Swallowing
		Diagnosis
			What Is the Differential Diagnosis for  a Thyroid Mass?
			What Is This Patient’s Diagnosis?
		History and Physical
			What Are the Common Symptoms of a Patient with a Thyroid Nodule?
			How Common Are Thyroid Nodules, and How Often Are They Cancerous?
			Describe the Different Types of Thyroid Cancer?
			What Are the Risk Factors for Thyroid Cancer?
			What Are the Symptoms of Hyperthyroidism?
			What Are the Symptoms of Hypothyroidism?
			How Should the Thyroid Be Examined?
			What Is the Significance of the Mass Moving Up and Down with Swallowing?
			What Is the Appearance of a Patient with Severe or Long-Standing Hyperthyroidism?
			What Is the Appearance of a Patient with Severe or Long-Standing Hypothyroidism?
		Pathophysiology
			What Is the Function of the Thyroid Gland?
			What Is Thyroglobulin?
			What Are the Actions of Thyroid Hormones?
			What Is the Embryologic Origin of the Thyroid?
			Why Isn’t Fine-Needle Aspiration (FNA) Able to Diagnose Follicular Thyroid Carcinoma?
			What Is Considered a Well-Differentiated Thyroid Cancer?
			What Are Psammoma Bodies?
		Workup
			What Laboratory Tests Are Recommended for a Thyroid Mass?
			What Are the Appropriate Imaging Studies for a Thyroid Nodule?
			Is Routine Use of Nuclear Imaging Useful for an Isolated Thyroid Nodule?
			What Nodules Should Undergo FNA?
			What Is the Bethesda Reporting System for FNAs of Thyroid Nodules?
		Management
			What Is the Next Step in a Patient with an Inadequate/Nondiagnostic FNA?
			What Is the Next Step for a Benign FNA Result?
			What Is the Next Step for an FNA Which Reports AUS or FLUS?
			What Is the Next Step for an FNA Result of Suspected Follicular Neoplasm?
			What Is the Next Step if the Thyroid Lobectomy Reveals Follicular Cancer?
			What Are the Advantages of Total Thyroidectomy in Thyroid Cancer Versus Lobectomy?
			What Thyroid Malignancy Is Amenable to Thyroid Lobectomy Alone? When Is Total Thyroidectomy Appropriate?
			Is It Ever Appropriate to Proceed Directly to Total Thyroidectomy Without Performing a Lobectomy First?
			What Is the Management of Anaplastic Thyroid Cancer?
			When Should Central Lymph Node Dissection Be Done? Lateral Neck Dissection?
			What Is the Role of Intraoperative Frozen Section Pathology?
			Which Thyroid Cancers Concentrate Radioactive Iodine?
			What Are the Important Elements of Postoperative Management of Thyroid Cancer?
				Radioactive Iodine Ablation
				Suppressive Thyroxine Therapy
		Follow-Up
			If You Suspect Recurrence, What Study Do You Perform?
			Does External Beam Radiation Play a Role in Thyroid Cancer?
			What Nonoperative Management Is Available for Thyroid Nodules?
		Complications
			What Are the Major Structures That Can Be Injured During Thyroidectomy?
			What Should Always Be Assessed Preoperatively in Patients Undergoing Thyroid Surgery that Have Had Previous Neck Surgery?
			What Is the Next Step if a Postoperative Patient Develops Stridor in the Recovery Room and the Neck Wound Appears to Be Tense?
		Key Areas Where You Can Get in Trouble
			Incomplete Family History in Patient with Thyroid Mass
			Obtaining CT with Iodinated IV Contrast in Patients with Thyroid Cancer
			Missing Postoperative Hypocalcemia
		Areas of Controversy
			Overdiagnosis of Thyroid Cancer
			Role of Prophylactic Central Neck Dissection (CND) for Papillary Thyroid Cancer
		Summary of Essentials
			History and Physical
			Diagnosis
			Pathophysiology
			Management
			Watch Out
		Suggested Reading
IV: Question Set: Endocrine
	Questions
	Answers
V: Head and Neck
	15: Progressively Hoarse Voice
		Diagnosis
			What Is the Differential Diagnosis for Hoarseness?
			What Is the Most Likely Diagnosis in This Patient?
		History and Physical Examination
			What Information Can the Nature of Hoarseness Provide?
			At What Point Should Hoarseness Warrant Consultation with an Otolaryngologist?
			Is History of Tobacco or Alcohol Use Important to Know?
			What Is the Significance of Bloody Sputum?
			What Is the Most Common Type of Laryngeal Cancer?
		Pathophysiology
			What Is the Innervation of the Larynx?
			Why Is Tobacco Use a Risk Factor for the Development of Squamous Cell Carcinoma of the Head and Neck?
			What Are the Potential Consequences of Unrecognized Squamous Cell Carcinoma of the Upper Aerodigestive Tract?
			Do Laryngeal Papillomas Cause Cancer?
		Workup
			What Examination Should Be Performed to Evaluate the Vocal Cords?
			If Laryngeal Cancer Is Suspected, What Additional Tests Are Warranted?
			What Is the Staging for Laryngeal Cancer?
		Management
			What Treatments Are Available?
			What Variables Affect Prognosis for Laryngeal Cancer?
		Areas Where You Can Get in Trouble
			Chronic Laryngitis as a Red Flag
			Airway Compromise
		Summary of Essentials
			History and Physical Examination
			Etiology/Pathophysiology
			Diagnosis
			Management
			Watch Out
		Suggested Reading
	16: Lump on the Neck Increasing in Size
		Diagnosis
			What Is the Differential Diagnosis? (Apply the “KITTENS” Mnemonic)
			What Is the Most Likely Diagnosis for This Patient?
			What Risk Factors Are Associated with Head and Neck Cancer in General?
			What Risk Factors Are Associated with Specific Head and Neck Cancers?
			How Have the Demographics of Head and Neck Cancer Changed in the Last 10 Years?
			What Premalignant Lesions Should Be Looked for on Physical Examination?
			What Symptoms Can Be Associated with Head and Neck Cancer, and What Are the Likely Sources?
			What Are the Key Aspects of the Head and Neck Exam in the Evaluation of a Solitary Neck Mass?
		Etiology
			What Is the Differential Diagnosis for Salivary Glands Tumors? What Is Their Malignant Potential?
			What Is Virchow’s Node? Why Is It Concerning?
			What Are the Most Common Sites of Head and Neck Cancer?
			What Is Meant by the Term “Primary Tumor”? What Is Meant by “Unknown Primary”?
			What “Primary Tumors” Arise in the Neck?
			What Does a Newly Discovered Malignant Neck Mass Most Likely Represent?
			What Types of Abscess Are in the Differential of a Neck Mass?
		Etiology/Pathophysiology
			What Is the Pathophysiology of Head and Neck Cancer? What Is Field Cancerization?
			What Is the Most Common Pathology of Head and Neck Cancer?
		Work-Up
			Is Observation an Acceptable Strategy for a Newly Discovered, Isolated, and Enlarged Cervical Lymph Node?
			In Addition to the Physical Exam Mentioned in the Vignette, What Procedure Is Performed in the Office by the Head and Neck Surgeon When a Metastatic Neck Lymph Node Is Suspected?
			Following This Procedure, What Are the Next Steps in the Work-Up of a Neck Mass?
			What Laboratory Tests Should Be Obtained?
			What Initial Imaging Modality Is the Diagnostic Test of Choice to Search for the Primary Tumor?
			What if the Initial Head and Neck CT Scan Fails to Demonstrate the Primary Tumor, What Additional Imaging Is Recommended?
			What Is the Best Way to Obtain a Tissue Sample to Determine if the Neck Mass Is a Metastatic Lymph Node?
			Once FNA Confirms that the Neck Mass Is a Metastasis, What Is the Next Step in Determining the Location of the Primary Tumor?
			What Is the Role of an Open Neck Biopsy in the Evaluation of Solitary Neck Mass?
			What Laboratory Studies Can Be Utilized to Assess for Infectious and Inflammatory Causes of Neck Masses?
		Management
		Areas Where You Can Get into Trouble
			Missing a Cancer Diagnosis
			Not Recognizing Potentially Life-Threatening Signs in Advanced Stages of Cancer
			Inappropriate Biopsy
		Areas of Controversy
			The Role of Whole-Body PET in the Management of Head and Neck Cancer
			The Role of Open Biopsy
		Summary of Essentials
			History and Physical
			Etiology/Pathophysiology
			Diagnosis
			Watch Out
		Suggested Reading
	17: Aural Fullness, Hearing Loss, and Tinnitus
		Diagnosis
			What Is the Differential Diagnosis for Hearing Loss in a Child?
			What Other Diagnoses Need to Be Considered in an Adult with Hearing Loss?
			What Is the Most Likely Diagnosis?
		History and Physical
			What Is the Peak Age for OME?
			What Are the Risk Factors for OME?
			What Is the Implication of Regression in Language?
			What Is the Implication of the Presence or Absence of Otalgia?
			What Is the Importance of Otorrhea?
			How Does One Distinguish Between Otitis Externa and Otitis Media on History and Physical Exam?
			What Are Some Abnormal Features Found on Otoscopic Examination that May Help Distinguish AOM from OME?
			What Is the Main Symptom Seen with OME?
			The Majority of OME Cases Spontaneously Resolve Within What Period of Time?
		Etiology/Pathophysiology
			What Is the Most Likely Etiology for Sudden Deafness?
			What Are the Two Main Causes of OME?
			What Other Diagnoses Need to Be Considered in an Adult with OME?
			What Is the Role of the Eustachian Tube (ET), and How Does It Differ Between Adolescents and Adults?
			Which Pathogens Are Most Commonly Found in OME?
			What Is the Most Significant Complication that May Result from OME?
		Workup
			What Does a Pneumatic Otoscopy Allow the Clinician to Do? How Is It Used?
			What Two Tests Will Help Differentiate a Conductive Hearing Loss from a Sensorineural Hearing Loss? How Are They Performed?
		Management
			In Children, What Is the Management for the Majority of Cases of OME?
			What Is Autoinflation? Is There a Role for It in OME?
			Are Antihistamines and/or Decongestants Recommended for OME in Children? How About Steroids or Antibiotics?
			Should Antibiotics Be Used Routinely for Patients with OME?
			What Are the Indications for Tympanostomy Tube Insertion?
			How Does the Management of OME Differ in Adults?
			What Are the Long-Term Complications from Untreated Otitis Media?
		Summary of Essentials
			History and Physical
			Pathophysiology
			Workup
			Management
			Complications
			Watch Out
		Suggested Reading
V: Question Set:
Head and Neck
	Questions
	Answers
VI: Hepatopancreaticobiliary
	18: Postprandial Right Upper Quadrant Pain
		Diagnosis
			What Is the Differential Diagnosis?
			What Is the Most Likely Diagnosis?
		History and Physical
			Why Is the Term Biliary Colic a Misnomer? What Is a Better Term?
			Why Is It Important to Distinguish Between Symptomatic Cholelithiasis and Acute Cholecystitis?
			How Does One Clinically Distinguish Between Symptomatic Cholelithiasis and Acute Cholecystitis?
		Pathophysiology
			What Is the Significance of Abdominal Pain After Eating Fatty Foods?
			What Is the Significance of RUQ Pain Combined with Scapular Pain?
			What Is the Significance of the Patient’s Inspiration Stopping with RUQ Palpation?
			What Is the Difference Between Visceral and Somatic Pain?
			What Is the Clinical Significance of the Patient’s Low-Grade Fever, Tachycardia, and Leukocytosis?
			What Is Chronic Cholecystitis?
			What Exactly Causes Acute Cholecystitis?
			What Is Hydrops of the Gallbladder?
			What Are the Typical Pathogens in Bile?
			What Are the Components of Bile?
			What Are the Two Main Types of Gallstones?
			What Are the Main Risk Factors for Developing Cholesterol Gallstones?
			How Do Cholesterol Gallstones Form?
			How Do Pigmented Gallstones Form?
			What Are the Different Manifestations of Gallstone Disease?
		Work-Up
			What Is the Next Step in the Work-Up? What Are the Specific Findings That Would Confirm the Diagnosis?
			What Is a Sonographic Murphy’s Sign?
			What Is the Normal CBD Diameter, and What Is the Implication of a Dilated CBD?
			How Accurate Is Ultrasonography in Detecting Gallstones Within the Gallbladder? Within the CBD?
			What If the Ultrasound Demonstrates Gas Bubbles in the Gallbladder Wall?
			Why Should Liver Tests, Amylase, and Lipase Always Be Sent in the Presence of RUQ and Epigastric Pain? What Is the Significance of Abnormalities?
			Is Amylase or Lipase Better to Rule Out Pancreatitis?
			What If Acute Cholecystitis Is Suspected but the Ultrasound Does Not Demonstrate Gallstones?
			Can You Develop Acute Cholecystitis Without Gallstones?
		Management
			What Is the Difference Between an Urgent and Emergent Case?
			What Is the Next Step in the Management of a Patient with an Ultrasound Demonstrating Gallstones, Pericholecystic Fluid, Gallbladder Wall Thickening of 5 mm, and a Positive Sonographic Murphy’s Sign?
			What Is the Ideal Choice of Antibiotics?
			What If Gallstones Are Discovered Incidentally? Do They Require a Cholecystectomy?
			How Should Symptomatic Cholelithiasis Be Managed?
		Postoperative
			What Is a Major Complication of Laparoscopic Cholecystectomy That Is More Common in the Setting of Acute Cholecystitis?
			What Is the Differential Diagnosis and Work-Up for Abdominal Pain That Develops Soon After Cholecystectomy?
			What Is the Differential Diagnosis If the Patient Develops RUQ Pain Several Weeks or Months After Cholecystectomy?
			What Is Sphincter of Oddi Dysfunction (SOD)?
		Area of Controversy
			When Should Acute Cholecystitis Be Managed Nonoperatively?
		Summary of Essentials
			History
			Physical Exam
			Pathology/Pathophysiology
			Diagnosis
			Management
			Postoperative
			Additional Important Facts
		Suggested Reading
	19: Right Upper Quadrant Pain, Fever, Nausea, and Vomiting
		Diagnosis
			What Is the Differential Diagnosis?
			What Is the Diagnosis for This Patient?
			What Are the Diagnostic Criteria for Cholangitis?
		History and Physical
			What Are the Causes of Obstructive Jaundice That Lead to Cholangitis?
			Why Does the Patient Have Dark Urine but Not Pale Stools?
			At What Level of Bilirubin Will Jaundice First Be Visible?
			Where Do You Look for Jaundice?
			What Is Charcot’s Triad?
			What Percent of Patients with Cholangitis Have All Three of the Triad?
			What Is Reynold’s Pentad? What Percent of Patients with Cholangitis Have All Components?
		Pathophysiology
			Why Are Gallstones the Most Common Cause of Obstructive Jaundice with Cholangitis?
			What Are Potential Consequences of Unrecognized Acute Cholangitis?
			Is Clinical Jaundice Always a Sign of Biliary Disease?
		Workup
			Why Is it Important to Distinguish Between Hepatic and Posthepatic Causes of Jaundice? How Do You Utilize the LFTs to Do So?
			What Imaging Is the Diagnostic Test of Choice? Does the Test Provide Direct or Indirect Evidence for the Diagnosis?
		Management
			What Are the Most Important Immediate Management Steps Once a Patient Develops Fever and Leukocytosis?
			What Is the Antibiotic Treatment for Acute Cholangitis?
			Does the Patient Need Admission to the ICU?
			Once the Patient Is Resuscitated, Antibiotics Started, and the Diagnosis Is Established, What Intervention Is Recommended? What Is the Goal of That Intervention?
			Following Successful ERCP, What Is the Next Management Step?
		Areas Where You Can Get in Trouble
			A History of Bloody Diarrhea in a Patient Who Presents with Cholangitis
			A Middle-Aged Woman with Pruritus, Fatigue, and Normal RUQ US
		Areas of Controversy
			Is Cholecystectomy Necessary After Biliary Decompression?
			Timing of ERCP with Severe Cholangitis: Emergently (in the Middle of the Night) or Urgently (the Next Morning)?
		Summary of Essentials
			History and Physical
			Etiology/Pathophysiology
			Diagnosis
			Management
			Watch Out
		Suggested Reading
	20: Severe Epigastric Pain with Nausea and Vomiting
		Diagnosis
			What Is the Differential Diagnosis for Epigastric Abdominal Pain?
			What Is the Diagnosis for This Patient?
			How Do You Diagnose Acute Pancreatitis?
		History and Physical
			What Nonsurgical Conditions Can Mimic an Acute Abdomen?
			What Is the Significance of Bruising Around the Umbilicus, Flank, and Inguinal Ligament?
			What Are the Signs, Symptoms, and Findings of Acute Pancreatitis?
			What Structures Are in the Retroperitoneum?
		Pathophysiology
			What Is the Pathophysiology of Pancreatitis?
			What Are the Etiologies for Pancreatitis?
			What Medications Can Cause Pancreatitis?
			How Do Gallstones Cause Acute Pancreatitis?
			In Patients with Gallstone Pancreatitis, How Often Does the Gallstone Remain Impacted in the Distal Common Duct?
			What Are the Differences Between Acute and Chronic Pancreatitis?
			How Is the Severity of Pancreatitis Classified?
			What Organ Systems Can Be Affected by Acute Pancreatitis?
			How Is Organ Failure Defined?
			What Is the Mechanism of Hypotension in Pancreatitis?
			What Are the Main Pulmonary Complications of Acute Pancreatitis?
			What Are the Different Histopathologic Types of Acute Pancreatitis? What Are the Important Differences?
		Prognosis
			How Is the Severity of Pancreatitis Determined?
			What Is the Main Drawback of Ranson Criteria?
			Why Does One Get Hypocalcemia with Severe Pancreatitis?
			What Is the Natural Disease Course of Acute Pancreatitis?
			What Is the Most Common Cause of Mortality in the First Week of Acute Pancreatitis? Beyond the First Week?
		Workup
			What Are the Most Important Laboratory Tests to Order when Suspecting Acute Pancreatitis?
			Which Laboratory Test Is Most Specific for Acute Pancreatitis?
			What Is the Diagnostic Imaging of Choice on Admission for Acute Pancreatitis?
			What Are the Classic Abdominal X-Ray Findings in Acute Pancreatitis?
			What Is the Classic Chest X-Ray Finding in Acute Pancreatitis? How Does This Finding Influence Prognosis?
			What Is the Role of Abdominal CT Scan on Admission?
			What Is the Role of Abdominal CT Scan During Subsequent Hospitalization?
			What Is the Role of Urgent ERCP in Gallstone Pancreatitis?
		Management
			What Is the Initial Treatment for Acute Pancreatitis?
			What Is the Subsequent Management Plan? And How Does This Differ Between Gallstone and Alcoholic Pancreatitis?
			How Does the Severity of Acute Pancreatitis Affect Management?
			What Is the Management Algorithm for Acute Pancreatitis? . Fig. 20.6
		Special Situations
			What Should You Suspect if a Patient with Severe Acute Pancreatitis Develops a Fever and Leukocytosis 4 Weeks into the Hospitalization?
			What Should You Suspect If a Patient with a Recent Hospitalization for Pancreatitis Comes in 6 Weeks Later with Vague Abdominal Pain, a Palpable Epigastric Mass, and Persistently Elevated Serum Amylase?
			What Are the Complications from Chronic Pancreatitis?
			Acute Pancreatitis After a Vascular Procedure
			What Is the Most Common Indication for Surgical Management in Chronic Pancreatitis?
		Areas of Controversy
			Is Urgent ERCP Beneficial for Severe Pancreatitis?
			Should Prophylactic Antibiotics Be Administered for Severe Acute Pancreatitis?
		Areas Where You Can Get in Trouble
			Missing Hypercalcemia as the Cause of Pancreatitis
			Pseudohyponatremia in Pancreatitis
			Nutritional Support
		Summary of Essentials
			History and Physical
			Pathophysiology
			Diagnosis
			Workup
			Management
			Complications
		Suggested Reading
	21: New Onset of Painless Jaundice
		Diagnosis
			What Is the Differential Diagnosis of Jaundice?
			What Is the Most Likely Diagnosis?
		History and Physical
			What Is Courvoisier’s Sign?
			What Is the Implication of Painful Versus Painless Jaundice?
			What Is Migratory Thrombophlebitis?
			What Are the Risk Factors for Pancreatic Cancer? Cholangiocarcinoma? Gallbladder Carcinoma?
			Where Is Jaundice Best Detected?
			What Is a Sister Mary Joseph Nodule? And What Is the Implication?
			What Is Blumer’s Shelf?
		Pathophysiology
			What Causes Jaundice?
			What Is the Metabolism of Bilirubin?
			What Is the Mechanism Behind “Clay-Colored” Stools?
		Workup
			In a Patient with Obstructive Jaundice, What Would the Typical Laboratory Findings Be?
			How Do You Distinguish Between Jaundice from Biliary Obstruction (Posthepatic) and Hepatocellular Damage?
			What Initial Imaging Is Recommended for Painful Jaundice?
			What Is the Recommended Imaging Choice for Painless Jaundice?
			What Is the Role of Magnetic Resonance Cholangiopancreatography (MRCP) and/or Endoscopic Retrograde Cholangiopancreatography (ERCP)?
			What Is the Role of Tumor Markers CA 19-9 and Carcinoembryonic Antigen (CEA)?
		Management
			What Criteria Make a Pancreatic Cancer Unresectable (Not a Surgical Candidate)?
			Should Patients Who Present with Obstructive Jaundice and a Resectable Pancreatic Mass Undergo Biopsy Confirmation?
			What Is the Role of Preoperative Stenting in the Presence of a Pancreatic Mass with Obstructive Jaundice?
			What Is the Role of Neoadjuvant Therapy for Pancreatic Adenocarcinoma?
			What Is the Surgical Management of Pancreatic Head (or Periampullary) Cancer?
			Why Must the Duodenum Be Removed When Surgically Resecting a Pancreatic Head Tumor?
		In Addition to Pancreatic Cancer, for What Other Conditions Is a Whipple Performed?
			What Are the Options for Palliation in Patients Who Are Not Candidates for Pancreatic Resection?
			What Is the Most Common Complication that Is Specific to a Whipple/Pancreatectomy? What Are Some Other Pancreas Surgery Specific Complications?
			Is There a High Risk of Diabetes After a Whipple Procedure?
			How Do You Suspect/Diagnose a Postoperative Pancreatic or Biliary Leak?
		Areas Where You Can Get in Trouble
			Increased INR (PT) with Pancreatic Cancer
			Incidental Liver Findings on CT Scan
		Area of Controversy
			Role of Adjuvant/Neoadjuvant Therapy in Pancreatic or Periampullary Cancer
		Summary of Essentials
			History and Physical
			Etiology/Pathophysiology
			Diagnosis
			Management
		Watch Out
		Suggested Reading
VI: Question Set: Hepatopancreaticobiliary
	Questions
	Answers
VII: Lower Gastrointestinal
	22: Bright Red Blood per Rectum
		Diagnosis
			What Is the Differential Diagnosis of a Lower  GI Bleed (LGIB)?
			What Is the Most Likely Diagnosis?
		History and Physical
			Why Is Age an Important Factor in a Patient with a LGIB?
			Why Are the Onset and Duration of Bleeding Important?
			What Do the Color and Amount of Blood Tell Us About the Source of Bleeding?
			What Is an Occult Bleed?
			What Associated Symptoms Are Important to Look for, and How Do They Help in the Differential Diagnosis?
			Why Is a History of Pelvic Radiation or Prior Aortic Surgery Important?
			Why Should One Inquire About Alcohol History and Look for Stigmata of Liver Cirrhosis?
			What Medications Can Exacerbate GI Bleeding?
			What Is the Implication of Abdominal Tenderness on Physical Examination?
			Can Upper and Lower GI Bleeds Be Distinguished Based on History and Physical Exam?
			What Is the Significance of Finding Iron-Deficiency Anemia in Association with a LGIB?
			How Does Ischemic Colitis Classically Present?
			What Are the Risk Factors for Diverticulosis?
		Etiology/Pathophysiology
			What Is a Diverticulum?
			What Causes a Diverticulum?
			What Is the Difference Between Diverticulum, Diverticulosis, and Diverticulitis?
			What Is the Most Common Site of Diverticula? Why?
			How Common Is a Diverticular Bleed?
			What Is the Natural History of a Diverticular Bleed?
			What Is Angiodysplasia? What Are the Risk Factors?
			What Causes Ischemic Colitis?
			What Factors Can Precipitate Ischemic Colitis?
			What Are the Primary Watershed Areas of the GI tract?
			What Is the Natural History of Ischemic Colitis?
			What Are the Differences Between Ischemic Colitis and Acute Mesenteric Ischemia?
		Initial Management
			What Are the Initial Steps in the Management of a LGIB?
			What Is the Next Step?
			How Does the Hemodynamic Stability of the Patient Affect the Subsequent Management and Diagnostic Workup?
			What Is the First Diagnostic Test of Choice in an Unstable Patient? How Effective Is This Test in Visualizing the Source of LGIB in the Acute Setting?
			What If This Diagnostic Test Is Not Able to Visualize the Bleeding? What Are the Two Options for Localizing the Source of Bleed? What Are the Advantages/Disadvantages of Each Modality?
			What Is the Next Step If These Modalities Fail to Identify the Exact Source of Bleeding?
			What If Arteriography Localizes the Source of Bleeding But Is Unable to Stop the Bleeding with Embolization?
		Areas Where You Can Get In Trouble
			Bleeding After Colonoscopic Biopsy or Polypectomy
			Pneumoperitoneum After Colonoscopy
		Summary of Essentials
			History and Physical
			Pathophysiology
			Management
		Suggested Reading
	23: Right Lower Quadrant Abdominal Pain
		Diagnosis
			What Is the Differential Diagnosis for Acute Appendicitis in an Adult? How Can Other Diagnoses Be Confused with Appendicitis?
			What Other Conditions Should You Consider in the Differential Diagnosis for Acute Appendicitis in Women?
			What Other Conditions Should You Consider in the Differential Diagnosis for Acute Appendicitis in a Child?
			What Is the Most Likely Diagnosis?
		History and Physical
			What Is Usually the First Symptom of Appendicitis and What Is the Classic Sequence of Symptoms?
			What Is the Significance of Absent Bowel Sounds?
			What Is a Hamburger Sign?
			What Are Rovsing’s, Psoas, and Obturator Signs and McBurney’s Point Tenderness?
		Pathophysiology
			What Explains the Transition From Periumbilical Pain to Right Lower Quadrant Pain in Appendicitis?
			Is the Appendix Considered Foregut, Midgut, or Hindgut? And How Does that Influence Where the Visceral Pain in the Abdomen Is Perceived?
			Can Appendicitis Present with No Abdominal Pain?
			Why Is Hyperesthesia of the Skin a Sign of Acute Appendicitis?
			What Is a Closed-Loop Obstruction?
			How Does a Closed-Loop Obstruction Pertain to Acute Appendicitis?
			What Are Some Other Examples of a Closed-Loop Obstruction?
			What Are Other Causes of Appendiceal Obstruction?
		Workup
			What Are the Critical Laboratory Values Utilized in the Workup of Acute Appendicitis?
			What Is the Significance of WBCs in the Urine Without Bacteria? How Might this Mislead the Clinician?
		What Further Imaging Is Needed?
			When Would Imaging Be Indicated? How Should the Use of Imaging Studies Differ Between Adults and Kids? Men and Women?
			Do Any Clinical Tools Exist to Help Decide if Acute Appendicitis Is Likely?
			What Are the Radiographic Signs of Appendicitis?
		Management
			What Is the Definitive Treatment for Appendicitis?
			Is Laparoscopic or Open Appendectomy the Superior Approach?
			What Is the Role of Pre- and Postoperative Antibiotics for Acute Non-perforated Appendicitis? For Perforated Appendicitis?
			How Should You Proceed if You Are Performing a Laparoscopic Appendectomy and You Discover that the Appendix Appears to Be Normal? Do You Remove the Appendix Anyway? Are There Circumstances Where an Appendectomy Is Contraindicated?
		Areas Where You Can Get in Trouble
			All Patients
				Misdiagnosing Appendicitis as Cystitis
				Misdiagnosing Other Causes of Perforation as Appendicitis
				Pseudoappendicitis
			Women
				Missing Ovarian Torsion: A Surgical Emergency
				Misdiagnosing Ruptured Appendicitis as Pelvic Inflammatory Disease (PID)
				Misdiagnosing Acute Appendicitis During Pregnancy
			Extremes of Age
				Under Age 5, Harder to Get History and Examine, and Are More Prone to Early Rupture
				Elderly: More Prone to Rupture
					Immunosuppressed Patient
		Areas of Controversy
			Is Appendicitis a True Surgical Emergency?
			Can Acute Non-perforated Appendicitis Be Managed Nonoperatively?
			Patients with Protracted History of Acute Appendicitis and a Palpable Abdominal Mass
			After Nonoperative Management of an Appendiceal Abscess, Is Interval Appendectomy Necessary?
		Summary of Essentials
			History
			Physical Exam
			Laboratory
			Diagnosis
			Imaging
			Pathophysiology
			Management
			Special Situations
		Suggested Reading
	24: Pencil-Thin Stools and Intermittent Constipation
		Diagnosis
			What Is the Differential Diagnosis for a Change in Bowel Habits?
			What Is the Most Likely Diagnosis?
		Screening
			What Screening Is Recommended for Colorectal Cancer?
			What Screening Is Recommended for Colorectal Cancer in Patients with a First-Degree Family Member with Colorectal Cancer? How About for IBD?
			What Are the Differences Between the Various Screening Modalities?
		History and Physical
			What Is the Significance of the Patient’s Unintended Weight Loss?
			What Are the Risk Factors for Colon Cancer?
			Where Does Colon Cancer Rank in Terms of the Most Common Cancers in the USA? In Terms of the Highest Overall Mortality?
			Are Right- or Left-Sided Colon Cancers More Common, and How Do the Presentations Differ?
			Why Is the Rectal Examination Important in Suspected Colorectal Cancer?
		Pathophysiology
			What Is a Polyp, and How Are They Generally Classified?
			What Is the Difference Between ­Hyperplastic and Hamartomatous Benign Polyps?
			How Do Colon Cancers Develop?
			What Features of an Adenoma Are ­Associated with an Increased ­Malignant Risk?
			Why Screen Every 10 Years When Using Colonoscopy?
			Why Is Colonoscopy Superior to Flexible Sigmoidoscopy for Screening?
			Where Are the Most Common Metastatic Sites for Colorectal Cancer?
			What Is the Relationship of Colon Cancer with Lynch Syndrome?
			What Are the Modified Amsterdam Criteria for HNPCC?
			What Is the Relationship of Colon Cancer with Familial Adenomatous Polyposis?
			What Are the Two Variants of FAP?
			What Is the Difference Between Synchronous and Metachronous Tumors?
		Workup
			For the Patient Described Above, What Would Be the Next Steps in the Workup?
			Is There Any Role for Testing His Stool for Blood?
			Once a Diagnosis of Colon Cancer Is Established, What Laboratory Tests Are Recommended?
			Is CEA Useful for Screening? How Is It Used?
			What Other Conditions Can Elevate the CEA Level?
			What Additional Imaging, if Any, Is Recommended Once a Diagnosis of Colon Cancer Is Established?
			How Is the Workup of Colon and Rectal Cancers Different?
			How Do You Stage Colon Cancer?
		Management
			How Are Polyps that Are Found During Colonoscopy Managed?
			What Is a “Bowel Prep,” and Why Is It Done?
			What Are the Recommended Operations in Colon Cancer?
			What Operations Are Typically Offered to Patients with Rectal Cancer?
			How Many Lymph Nodes Should Be Resected?
			Why Is Neoadjuvant Therapy Used for Rectal Cancer and Not Colon Cancer?
			Why Is Radiation Used for Rectal Cancer and Not Colon Cancer?
			What Are the Major Complications Specific to Colon Surgery?
		Areas of Controversy
			Treatment of Stage-IV (metastatic) Colon Cancer
			Is It Useful to Follow CEA Levels After Surgery?
		Summary of Essentials
			History and Physical
			Screening
			Pathophysiology
			Diagnosis
			Workup
			Management
			Complications
		References
	25: Chronic Constipation Presenting with Severe Abdominal Pain
		Case Study
		Diagnosis
			What Is the Differential Diagnosis?
			What Is the Most Likely Diagnosis?
		History and Physical
			What Is the Difference Between Obstipation and Constipation?
			What Are the Classic Physical Exam Findings in Bowel Obstruction?
			What Clues on History and Physical Examination Help Distinguish Between Small and Large Bowel Obstruction
			Why Is a History of Neurologic or Psychiatric Disorders Important?
			What Is the Classic Presentation for Ogilvie’s Syndrome?
			What Are the Five Fs of Abdominal Distention?
		Etiology/Pathophysiology
			What Are the Most Common Causes of LBO?
			Where in the Colon Is a Cancer Most Likely to Cause an LBO?
			What Is the Difference Between Malrotation and Volvulus?
			How Does the Etiology Differ Between Sigmoid and Cecal Volvulus?
			What Are the Risk Factors for Sigmoid Volvulus?
			What Is Meant by the Term Complicated Volvulus?
		Workup
			What Are the First Steps in the Workup of a Suspected Large Bowel Obstruction?
			What Is the First Imaging Recommended for a Suspected Large Bowel Obstruction?
			How Is Cecal Volvulus Differentiated from Sigmoid Volvulus on X-Ray?
			What Additional Imaging Is Recommended?
		Management
			What Are the Initial Steps in the Management of a Large Bowel Obstruction at Any Anatomic Location?
			What Is the Subsequent Treatment for a Sigmoid Volvulus?
			Following Successful Detorsion of Uncomplicated Sigmoid Volvulus, What Is the Next Step in the Management?
			How Is Complicated Sigmoid Volvulus Managed?
			What Is the Treatment for a Cecal Volvulus?
			What Factors Determine Outcomes of Volvulus?
		Area Where You Can Get in Trouble
			LBO in Pregnancy
		Area of Controversy
			Percutaneous Endoscopic Colostomy Tubes
		Summary of Essentials
			History and Physical
			Etiology/Risk Factors
			Diagnosis
			Management
		Suggested Reading
	26: Left Lower Quadrant Pain and Fever
		Diagnosis
			What Is the Differential Diagnosis?
			What Is the Most Likely Diagnosis?
		History and Physical
			What Are the Risk Factors for Diverticulosis?
		Pathophysiology
			Where in the Colon Do Diverticula Occur Most Frequently? Which Diverticula Are More Prone to Infection? Bleeding?
			Do Diverticula Occur in the Rectum? Why/Why Not?
			What Are True and False Diverticula? Which Type Are Sigmoid Diverticula?
			What Are the Main Complications of Diverticulitis? How Would They Present?
			What Are the Main Etiologies for a Colovesical Fistula? Colovaginal Fistula?
			How Does a Colovesical Fistula Present?
			What Is Meant by Complicated Versus Uncomplicated Diverticulitis? What Is the Implication?
			What Is the Hinchey Staging System?
		Workup
			What Imaging Is Recommended?
			What Two Diagnostic Studies Are Contraindicated in the Acute Setting of Suspected Diverticulitis and Why?
		Management
			What Is the First Step in the Management of Suspected Acute Diverticulitis?
			What Is the Subsequent Management for Uncomplicated Diverticulitis?
			What Operation Is Performed in the Elective Setting for Sigmoid Diverticulitis? How Is This Operation Different than Surgery in the Urgent or Emergent Setting?
			Complicated Diverticulitis
				What Is the Management of Complicated Diverticulitis?
				What Are the Indications for Urgent Surgery? What Operation Is Recommended in the Urgent Setting?
				How Does One Determine the Proximal and Distal Extent of Colon Resection? What if There Are Diverticula Throughout the Colon?
				What Is the Recommended Treatment for Diverticulitis Complicated by a Localized Abscess?
				What Is the Recommended Treatment for Diverticulitis with a Colovesical Fistula?
				Is the Presence of Free Air on CT an Absolute Indication for Urgent Surgery?
		Areas Where You Can Get in Trouble
			Not Identifying the Ureter During Sigmoid Colon Resection
			Elderly Patient with Chronic Constipation and NSAID Use, with Sudden Abdominal Pain
		Areas of Controversy
			How Many Episodes of Uncomplicated Diverticulitis Are Acceptable Before Elective Surgery Is Recommended?
			Benefit of Bowel Prep Before Elective Colon Surgery
		Summary of Essentials
			History and Physical
			Pathophysiology
			Diagnosis
			Management
		Suggested Reading
	27: Fatigue and Bloody Diarrhea
		Diagnosis
			What Is the Differential Diagnosis of Chronic Diarrhea with Mucus and/or Blood?
			What Are the Key Differences Between Ulcerative Colitis and Crohn’s?
			What Is the Diagnosis for This Patient?
		History and Physical
			What Is the Age Distribution of Inflammatory Bowel Disease (IBD)?
			What Are the Risk Factors for IBD?
			Is Smoking a Risk Factor for Both Crohn’s and UC?
			How Do Patients with IBD Typically Present Clinically?
			What Are the Extraintestinal Manifestations of IBD?
			What Is the Significance of Growth Retardation and Delayed Onset of Puberty?
			What Is Tenesmus?
		Pathophysiology
			What Is the Final Common Pathway for the Pathogenesis of IBD?
			Are Patients with IBD at Increased Risk for Cancer?
			What Is the Screening Recommendation for Colon Cancer in IBD?
			What Portion of the GI Tract Is Most Commonly Affected in Crohn’s?
			What Complications Affect IBD Patients?
			What Types of Anemia Are Seen with IBD?
			What Other Nutritional Deficiencies May IBD Patients Have?
			What Conditions Will Prevent Fistulas from Closing?
		Workup
			What Laboratory Studies Are Recommended? What Serology Is Recommended?
			What Is the Primary Modality for Diagnosing IBD?
			What Are the Classic Findings on CT Scan?
		Management
			What Are the Medical Therapies Available?
			What Percentage of IBD Patients Will Eventually Require Surgery?
			What Are the Surgical Indications for Crohn’s? Ulcerative Colitis?
			Why Is It Recommended to Avoid Small Bowel Resection with Crohn’s Disease if Possible?
			What Are Complications of Removing the Terminal Ileum?
			In a UC Patient, What Would Be the Concern if They Presented with Sepsis, Diffuse Abdominal Pain, and a Markedly Distended Colon?
		Areas Where You Can Get in Trouble
			Flank Pain and Hematuria in Crohn’s
			Cecal or Terminal Ileum Inflammation in Association with Suspected Appendicitis
		Summary of Essentials
			History and Physical
			Pathophysiology
			Diagnosis
			Management
		Suggested Reading
VII: Question Set: Lower
Gastrointestinal
	Questions
	Answers
VIII: Neurosurgery
	28: Neck Pain and Extremity Weakness Following Trauma
		Diagnosis
			What Is the Differential Diagnosis for Cervical Spine Injury?
			What Is the Most Likely Diagnosis?
		History and Physical
			What Are the Most Common Cervical Spinal Levels Involved After Trauma?
			What Dermatome Level Supplies the Shoulders? What Must You Remember About the Dermatome Map When Testing Sensation on the Chest?
			Why Is It Important to Check Deep Tendon Reflexes?
			How Are Deep Tendon Reflexes Graded?
			Clinically, What Is the Difference Between a Complete and Incomplete Spinal Cord Injury?
			What Are the Devastating Clinical Examination Findings in Patients with Complete Spinal Cord Injury in the High Cervical Cord (at or Above C3)?
			What Is the Term for Sensory or Motor Dysfunction Caused by Pathology of a  Nerve Root? What Are the Clinical Signs and Symptoms Associated with This Disorder?
			What Is the Term for Sensory or Motor Dysfunction Caused by Pathology of the Spinal Cord? What Are the Clinical Signs and Symptoms Associated with This Disorder?
		Pathophysiology
			What Is the Difference Between Spinal Shock and Neurogenic Shock?
			In the Context of Trauma, Why Are Thoracic Spine Injuries Less Common Than Cervical Spine Injuries?
			What Is Sacral Sparing? What Is Its Significance in the Setting of Spinal Shock?
			What Are the Common Mechanisms for Neck Injuries?
			What Is Atlanto-Occipital Dislocation?
			What Is the Pathophysiology of the Three Most Common Incomplete Spinal Cord Injuries?
			Why Are the Distal Lower Extremities Typically Spared in Central Cord Syndrome?
			Which Incomplete Cord Syndrome Carries the Best Prognosis for Recovery of Neurological Function, Bowel and Bladder Function, and Ambulatory Capacity?
			Which Incomplete Cord Syndrome Carries the Worst Prognosis for Functional Recovery?
			What Spinal Cord Injury Can Occur After Surgery on the Thoracic Aorta?
		Workup
			How Do You Diagnose Spinal Cord Injuries and Cervical Vertebral Fractures?
			What Are the NEXUS Criteria?
			What Type of Imaging Should Be Obtained?
			What if CT Scan Is Not Available? What About Children?
			What Is the Indication for Ordering an MRI Scan of the Cervical Spine?
			What Is SCIWORA and SCIWOCTET?
		Management
			What Are the General Treatment Principles for Patients with Cervical Spine Injury?
			How Is Neurogenic Shock Treated?
			What Are the Goals of Management for Patients with Complete Spinal Cord Injuries or High Cervical Cord Injuries?
			Why Is There a Lower Threshold for Surgical Intervention in Patients with Incomplete Spinal Cord Injury Versus Patients with Complete Spinal Cord Injury?
			What Are the General Indications for Emergent Surgery with Spinal Cord Injury?
		Area of Controversy
			Steroid Therapy for Acute Spinal Cord Injury
		Areas Where You Can Get in Trouble
			Complications with the Exposure of the Anterior Cervical Spine
			Impaired Upper Extremity Strength After Whiplash
		Summary of Essentials
			History and Physical
			Pathophysiology
			Workup
			Management
			Complications
		Suggested Reading
	29: Loss of Consciousness Following Head Trauma
		Diagnosis
			What Is the Differential Diagnosis for a Severe (GCS ≤8) Traumatic Brain Injury?
			What Is the Most Likely Diagnosis?
		History and Physical
			What Is the Definition of a TBI?
			How Do You Calculate the GCS?
			What Non-head Trauma Factors Affect GCS?
			What Are Raccoon Eyes? What Is Battle’s Sign?
			How Does the Physical Exam Help to Localize the Site of Intracranial Bleeding?
			What if the Blown Pupil and the Posturing Are on the Same Side? How Do You Use These Findings to Lateralize the Suspected Lesion?
			What Is the Implication of Abnormal Arm Flexion/Leg Extension with Pain ­Stimulation? Arm/Leg Extension?
			How Does the Presentation of Chronic SDH Different from Acute SDH?
		Pathophysiology
			What Is a Concussion?
			What Is Uncal Herniation?
			What Is the Pathophysiology of an Epidural Hematoma?
			What Is the Implication of a “Lucid” Interval with Head Injury?
			What Is the Pathophysiology of an Acute Subdural Hematoma?
			What Is the Formula for Cerebral Perfusion Pressure?
			What Other Factors Affect Cerebral Perfusion Pressure?
			What Is the Most Powerful Intracranial Vasodilator?
			What Is the Pathophysiology of Cushing’s Triad?
			What Is a Coup vs. Contrecoup Injury?
		Initial Management
			What Initial Management Is Recommended for the Above Patient?
			What Are the Indications for Head CT with TBI?
			Describe the Findings on Head CT Seen with EDH, SDH, and SAH Intraparenchymal Contusion
			What Are the Indications for ICP Monitoring?
			What Is the Role of Hyperventilation?
			What Other Medical Treatment Options Are There for Intracranial Hypertension?
			How Does Mannitol Work?
			Is There a Role for Corticosteroids in the Treatment of TBI? What Is the Difference Between Vasogenic and Cytotoxic Edema?
			When Should a Craniotomy Be Performed for Acute EDH or SDH? What About ­Craniectomy?
			What Are the Guidelines for Repeat ­Imaging if a Nonoperative Management Is Planned?
			What Are the Criteria for Brain Death? What Conditions Must Be Ruled Out?
			What Factors Affect Prognosis of Head Injury?
		Area You Can Get in Trouble
			Guidelines for Head CT in Children
		Summary of Essentials
			History and Physical
			Pathophysiology
			Initial Management
		Suggested Reading
VIII: Question Set:
Neurosurgery
	Questions
	Answers
IX: Orthopedic
	30: Multiple Extremity Injuries After Motorcycle Accident
		Diagnosis
			What Is the Most Likely Diagnosis?
		History and Physical
			What Should Be Assessed During Extremity Evaluation of a Trauma Patient?
			What Is an Open Fracture?
			What Is the Likely Etiology of the Neurologic Deficit in His Right Arm?
			What Is the Significance of Hearing a Bruit on Auscultation of the Right Clavicle?
			How Does Fat Embolism Syndrome Present?
			What Is a Dangerous Sequela for a Tibia Fracture (or Forearm Fracture)?
			What Concomitant Fracture Is Important to Consider in All Femur Fractures?
		Pathophysiology
			What are Seddon’s Three Basic Categories of Nerve Injury?
			How Fast Does an Injured Axon Regenerate?
			Does Wallerian Degeneration Occur with Neurapraxia?
			What Are the Three Layers of the Nerve Sheath?
			What Are the Classic Nerve Injuries Associated with Fractures?
			What Is the Presumed Pathophysiology of Fat Embolism Syndrome?
			What Artery Is Most at Risk with a Supracondylar Humerus Fracture?
		Workup
			What Is the Extent of Imaging Recommended with All Long Bone Fractures?
			What Is a Floating Knee?
		Management
			How Are Open Fractures Graded?
			What Is the Most Important Determinant of Severity for Fractures?
			What Antibiotics Are Indicated for Open Fractures in Farm Accidents or Soil-Contaminated Wounds?
			What Else Should Be Considered for Wounds Contaminated with Dirt and Soil?
			What Are the Principles of Surgical Management of an Open Fracture? What Is the Optimal Timing?
			How Are Skin Wounds Closed?
			Why Is It Important to Have Early Stabilization of Open Fractures?
			Which Should Be Repaired First in a Patient with Concurrent Bone Fracture and Associated Vascular Injury with a Hard Sign (e.g., No Distal Pulse)?
			What Is the Difference Between Open and Closed Reduction?
			What Is the Difference Between Internal and External Fixation? What Are the Advantages/Disadvantages?
			What Are the Main Management Concerns for a Femur Fracture?
			What Single Type of Orthopedic Fracture is at the Greatest Risk for Hemorrhagic Shock?
			What Is the Optimal Timing of Femur Fracture Repair? What Is the Main Risk if Repair Is Delayed? What Are the Main Options in Repair?
			Do All Femur Fractures Require Fixation?
			What Should Be Considered in a Child Who Is Not Yet Walking and Presents With a Femur Fracture?
			What Is the Management for Fat Embolism Syndrome?
			What Is the Most Important Factor to Prevent Fat Embolism Syndrome Before It Occurs in a Polytrauma Patient?
			What Is the Management for a Clavicle Fracture?
		Postoperative
			What Are the Important Risks Associated with Fracture Treatment?
		Area of Controversy
			Limb Salvage or Amputation
		Areas Where You Can Get in Trouble
			Missing Other Fractures
			Failure to Recognize a Compartment or Impending Compartment Syndrome
			Failure to Recognize a Neurologic Deficit
		Summary of Essentials
			History and Physical
			Pathophysiology
			Diagnosis
			Management
				Open Fracture
				Closed Fracture
			Postoperative
		Suggested Reading
	31: Immediate Swelling After Trauma to the Knee
		Diagnosis
			What Is the Differential Diagnosis for Acute Knee Pain?
			What Is the Most Likely Diagnosis?
		History and Physical Exam
			What Are the Principal Components of the Knee Exam?
			What Is the Classic History for the Various Ligamentous Injuries in the Knee?
			What Are the Classical Physical Exam Signs for Knee Injuries and How Are They Performed?
			What Does a History of Locking or Catching Signify?
			Why Is It Important to Perform a Careful Vascular Exam?
			Why Is It Important to Perform a Careful Neurologic Exam?
			What Is the Significance of Having Soft Muscular Compartments in the Lower Leg?
			What Is the Importance of Asking About Fever and Other Constitutional Symptoms Such as Fatigue, Weight Loss, Night Sweats, and Decreased Appetite?
			What Is the Importance of Asking About Previous Knee Surgery?
			What Should Be Considered in Patients with Recurrent Fractures?
			What Is the Significance of Being Able to Bear Weight?
			Why Is It Important to Look for Lacerations or Wounds?
		Pathophysiology
			What Two Basic Mechanisms Lead to ACL Injury?
			Is ACL Injury More Common in Men or Women?
			What Is the Terrible (aka “Unhappy”) Triad of the Knee?
			What Is a Common Method of Injuring the Medial Meniscus?
			What Is the Distribution of Blood Supply to the Meniscus and Why Is It Important?
			What Is the Significance of a “Popping” Noise on Traumatic Injury?
			What Injury Would You Expect from a Dashboard Knee Injury?
			What Is the Normal Range of Motion of the Knee?
			What Is the Significance of Being Unwilling (vs. Unable) to Range the Knee?
			What Is the Significance of Being Unable to Actively or Fully Extend the Knee While the Examiner Can Do So Passively?
			What About Pain Inferior to the Patella with an Intact Extensor Mechanism of the Knee?
			Why Is It Important to Examine the Patella and Its Tracking?
		Workup
			What Is the Next Step in the Workup of This Patient?
			What Radiographic Sign Is Pathognomonic for ACL Injury?
			What Is the Meaning of Patella Alta and Baja on an X-Ray?
			What Are the Characteristic Radiographic Features of Osteoarthritis of the Knee and Why Is It Important to Consider After ACL Injury?
			How Should Suspected Septic Arthritis or Crystalline Arthropathy Be Worked Up?
			What Imaging Would You Use to Confirm a Suspected ACL Injury?
		Management
			Is There a Role for Nonoperative Management of ACL Tears?
			What Are the Risks of Nonoperative Management of an ACL Injury?
			What Is the Management for the Patient in This Case?
			What Is the Management of a Meniscal Tear?
		Postoperative
			What Are the Main Risks Associated with ACL Surgery?
		Areas of Controversy
			What Are the Different Types of ACL Grafts Used?
			What About ACL Injuries in Adolescents Who Have Not Completed Their Growth?
		Areas Where You Can Get in Trouble
			Failure to Recognize a Knee Dislocation or Vascular Injury
			Failure to Recognize a Septic (Infected) Knee Joint
		Summary of Essentials
			Diagnosis
			History and Physical Exam
			Etiology/Pathophysiology
			Workup
			Management
			Postoperative
		Suggested Reading
	32: Adolescent Male with Right Groin Pain and Limp
		Diagnosis
			What Is the Differential Diagnosis for Groin Pain with a Limp or Refusal to Bear Weight?
			What Is the Most Likely Diagnosis?
		History and Physical
			What Important Demographic Factor Limits the Differential Diagnosis for This Patient?
			What Is an Antalgic Gait?
			What Risk Factors Are Shared by Patients That Develop SCFE?
			What Is the Most Common Presentation of SCFE and Physical Exam Findings?
			What Is the Significance of External Rotation of His Leg and Resisting Internal Rotation?
			What if the Patient Holds the Thigh in Slight Hip Flexion and Internal Rotation?
			Why Is It Important to Examine the Hip in the Setting of Knee Pain?
			Why Is It Important to Consider Delayed Growth, Puberty, or Endocrinological Signs or Symptoms if SCFE Is Suspected?
			What Is the Significance of Pain That Radiates into the Groin?
			Why Is It Important to Clarify the Meaning of Hip Pain on History?
			What Would Be the Significance of a History of a Recent Upper Respiratory Tract or Other Viral Infection?
			Why Is It Important to Ask About Travel or Exposure to Wildlife?
			Why Is a Family History of Joint Disease Important?
			Why Would a History of Corticosteroid Use Be Important?
			What Are Other Important Risk Factors for Secondary Osteonecrosis of the Hip?
			What Is the Significance of a Leg Length Discrepancy with This Clinical Presentation?
			Why Is It Important to Examine the Back?
			What Is the Importance of the Ability to Bear Weight or Range the Hip?
			What Are the Component Steps in the Orthopedic Examination of a Painful Hip?
			What Are Some Common Physical Exam Tests Utilized in the Examination of the Hip?
		Pathophysiology
			What Is the Pathophysiology for the ­Orthopedic Mechanical Diagnoses?
			Are “Shin Splints” the Same as Tibial Stress Fractures?
		Workup
			What Laboratory Tests Would Be Useful and Why?
			What Analysis Should Be Done if Joint Aspiration or Arthrocentesis Is Done for Suspected Infection?
			What Is the First Imaging That Would Be Recommended?
			What Are the Classic X-ray Findings of Pediatric Orthopedic Mechanical Hip Disease?
			What Other Imaging Is Useful if the  Diagnosis Is Unclear?
		Management
			What Is the Management for the Following Diagnoses on the Differential?
		Complications
			What Are the Important Potential Complications of Surgery for SCFE?
		Area of Controversy
			Do Both Hips Require In Situ Screw Fixation in Patients with SCFE?
		Area Where You Can Get in Trouble
			Never Miss Septic Arthritis
		Summary of Essentials
			History and Physical
			Pathophysiology
			Workup
			Management
			Complications
		Suggested Reading
	33: Chronic Right-Hand Pain
		Diagnosis
			What Is the Differential Diagnosis?
			What Clues on History and Physical Examination Might Direct You Toward Specific Diagnoses?
			What Is the Most Likely Diagnosis?
		History and Physical
			What Is the Most Common Nerve Entrapment Syndrome in the Upper Extremity?
			What Are the Major Risk Factors for CTS?
			What Are Key Components in the Hand History Portion of the Exam?
			What Are the Best Known Classic Signs of CTS?
			What Is Durkan’s Median Nerve Compression Test?
			Are There Any Other Overlapping Median Nerve Diagnoses to Consider?
			What Is Acute CTS?
			What Physical Exam Finding Helps Distinguish Proximal Median Nerve Compression at the Elbow (Pronator Teres Syndrome) from Compression at the Carpal Tunnel?
			What Findings Help Distinguish CTS from Cervical Spine Root Pathology?
			What Is Thoracic Outlet Syndrome and What Physical Exam Tests Distinguish It from Spinal Root Pathology?
			What About Myelopathy?
		Pathophysiology
			What Nerve Is Compressed in CTS?
			What Is the Motor Distribution of the Median Nerve in the Hand?
			What Is the Sensory Distribution of the Median Nerve in the Hand?
			What Is the Origin of the Median Nerve from the Cervical Spine Roots?
			Where Is the Median Nerve Located in the Wrist?
			What Are the Boundaries of the Carpal Tunnel?
			How Many Structures Course Through the Carpal Tunnel, and What Are the Contents of the Carpal Tunnel?
			What Is the Pathophysiology of CTS and Potential Causes?
			What Is the Significance of Thenar Wasting?
			What Are the Three Stages of Median Nerve Compression, and Why Are They Important?
			How Does the Presentation of a Median Nerve Lesion at the Elbow Differ from a Lesion at the Wrist?
			What About the Ulnar Claw?
			What Is Froment’s Sign?
			What Systemic Condition Is Often Associated with CTS?
		Workup
			How Is Carpal Tunnel Diagnosed?
			Do Any Medical Neuropathies Need to be Ruled Out?
			How is Suspected Cervical Spine Disease Worked Up?
		Management
			What Is the Initial Treatment for CTS?
			What Is the Second Line of Treatment?
			What Are Indications for Surgical Intervention for CTS?
			What Is the Gold Standard Treatment for CTS?
			How Is CTS Associated with Hypothyroidism Managed?
		Complications
			What Are the Complications of Carpal Tunnel Release?
		Areas of Controversy
			Endoscopic Versus Open Carpal Tunnel Release
			Need for EMG/NCS
		Areas Where You Can Get in Trouble
			What Is the “Million-Dollar Nerve”?
			Injury to the Palmar Cutaneous Branch
			Failure to Recognize Spinal Pathology
		Summary of Essentials
			History
			Physical Exam
			Pathophysiology
			Diagnosis
			Management
			Postoperative
			Additional Important Facts
		Suggested Reading
IX: Question Set:
Orthopedic Surgery
	Questions
	Answers
X: Pediatric
	34: Full-Term Male Infant with Respiratory Distress
		Diagnosis
			What Is the Differential Diagnosis of Surgical Causes of Neonatal ­Respiratory Distress
			What Is the Most Likely Diagnosis?
			What Are the Most Common Causes of Neonatal Respiratory Distress?
		History and Physical
			What Is the Significance of the Supracostal Retractions and Grunting?
			What Information Does the Pulmonary Examination Provide?
			Why Is the Heartbeat Displaced?
			Why Is the Absence of Prenatal Care Important?
		Pathophysiology
			What Is Thought to Be the Etiology of CDH?
			Are There Different Type of CDHs?
			How Is Fetal Blood Shunted Away from  Pulmonary Circulation In Utero?
			What Are the Changes That Occur During Childbirth That Allow the Neonate to Transition to Breathing Air?
			How Is This Transition Affected By CDH?
			Are There Associated Anomalies?
		Workup
			What Is the Best Initial Diagnostic Test?
		Management
			What Is the Most Important Step in Clinical Management?
			What Type of Ventilation Is Recommended? What if the Patient Cannot Be Adequately Oxygenated and/or Ventilated?
			What Is the Timing of Surgical Repair?
		Area Where You Can Get in Trouble
			Initial Intubation
		Area of Controversy
			Type of Repair: Open Versus Minimally Invasive Surgery (MIS)
		Prognosis
			What Is the Prognosis for CDH? What Are the Factors Affecting Prognosis?
		Summary of Essentials
			History, Physical Examination, and Diagnosis
			Etiology/Pathophysiology
			Management
		Suggested Reading
	35: Infant with Bilious Emesis
		Diagnosis
			What Is the Differential Diagnosis for Bilious Emesis in an Infant?
			How Does Age Affect the Differential Diagnosis of Bilious Emesis?
			What Is the Diagnosis?
		History and Physical
			Why Is It Important to Distinguish Between Bilious and Nonbilious Vomiting in an Infant?
		Workup
			What Is the First Imaging Study to Obtain?
			What Is the Next Step in Diagnosis?
		Pathophysiology
			What Defines the Midgut?
			What Is the Normal Developmental Sequence of Events of the Human Midgut?
			What Is the Etiology of This Condition?
			Does Malrotation Always Result in Midgut Volvulus? Is It Always Acute?
		Workup
			Are Plain Radiographs Always Necessary?
		Management
			What Is the Most Important Immediate Management Issue?
			What Operation Is Required?
		Area Where You Can Get in Trouble
			“Double-Bubble” Sign on Abdominal X-Ray in Neonate with Bilious Emesis
		Area of Controversy
			Infants with Complete Midgut Infarction
		Summary of Essentials
			History, Physical Examination, and Diagnosis
			Etiology/Pathophysiology
			Management
			Watch Out
		Suggested Reading
	36: Infant with Nonbilious Emesis
		Case Study
		Diagnosis
			What Is the Differential Diagnosis?
			What Is the Most Likely Diagnosis?
		History and Physical
			Why Is It Important to Distinguish Between Bilious and Nonbilious Vomiting in an Infant?
			What Is the Classic History for HPS?
			What Are the Classic Physical Examination Findings?
		Pathophysiology
			What Is the Pathophysiology of This Condition?
		Workup
			What Is the First Imaging Study to Obtain?
			What if the Diagnosis Is Still Uncertain?
			What Electrolyte Abnormalities Would You Expect?
		Management
			What Is the Most Important Immediate Management Issue?
			What Is the Treatment?
			What Is the Timing of Surgery?
			When Can the Patient Resume Eating?
		Areas Where You Can Get in Trouble
			Inadequate Resuscitation Prior to Surgery
			Postoperative Complications
		Area of Controversy
			Nonsurgical Management
		Summary of Essentials
			History, Physical Examination, and Diagnosis
			Pathophysiology
			Management
		Suggested Reading
	37: Infant Born with Abdominal Wall Defect
		Case Study
		Diagnosis
			What Is the Differential Diagnosis for a  Newborn with Abdominal Wall Defect and What Are the Common and  Distinguishing Features?
			What Is the Most Likely Diagnosis in This Case?
		History and Physical Examination
			Which of the Above Diagnoses Are the Most Common?
			What Are the Specific Differences Between Gastroschisis and Omphalocele?
			What Are the Risk Factors for Gastroschisis and for Omphalocele?
		Pathophysiology
			What Is the Etiology of Gastroschisis?
			What Is the Etiology of Omphalocele?
			Which Condition Is More Urgent? Why?
		Workup
			What Is the Next Step in the Workup?
		Management
			What Is the Neonatal Management of Gastroschisis and Omphalocele?
			What Is the Timing of the Closure of the Abdominal Wall Defect?
		Areas Where You Can Get in Trouble
			Not Properly Protecting the Bowel
			Not Addressing Nutritional Needs
			Not Recognizing Abdominal Compartment Syndrome After Repair
		Area of Controversy
			Method of Repair of Large Abdominal Wall Defect
		Summary of Essentials
			Diagnosis
			Omphalocele-Associated Conditions
			Etiology
			Neonatal Management
			Postoperative Care
		Suggested Reading
	38: Excessive Drooling in a Newborn
		Diagnosis
			What Is the Differential Diagnosis of Excessive Drooling/Feeding Intolerance in a Newborn?
			What Is the Most Likely Diagnosis?
		History and Physical
			What Are Other Possible Presentations of TEF?
		Pathophysiology
			What Is the Significance of Oxygen Desaturation that Only Occurs While Feeding?
			What Is Thought to Be the Etiology of EA?
			How Are the Different Types Classified?
			What Are the Associated Anomalies?
		Work-up
			What Is the Best Initial Diagnostic Test?
			Are Other Studies Necessary for Diagnosis?
			Can the Diagnosis of TEF Be Made Prenatally?
		Management
			What Is the First Step in Clinical Management?
			What Is the Timing of Surgical Repair?
			Are Any Other Studies Needed Prior to Surgery?
			What Are the Complications After Surgical Repair?
			What Is the Prognosis?
		Area Where You Can Get in Trouble
			Intubation and TEF
		Summary of Essentials
			Diagnosis
			History and Physical Examination
			Pathophysiology
			Management
		Suggested Reading
X: Question Set:
Pediatric Surgery
	Questions
	Answers
XI: Surgical Complications
	39: Postoperative Bleeding
		Case Study
		Diagnosis
			What Is the Differential Diagnosis of Bleeding in the Postoperative Setting?
			What Is the Most Likely Cause of the Bleeding in the Patient Described Above?
			What Is the Differential Diagnosis for Prolonged PTT and the Common Features?
			What Is the Most Likely Diagnosis?
		History and Physical Examination
			Why Is It Important to Ask About a History of Bleeding After Minor Trauma/Procedures?
			Why Is It Important to Ask About a Family History of Bleeding?
			What Medical Conditions Are Risk Factors for Bleeding?
		Physiology/Pathophysiology
			What Is the Difference Between Primary and Secondary Hemostasis Disorders?
			What Is Coagulopathy?
			What Is Meant by a Medical Versus a Surgical Postoperative Bleed?
			What Is the Pathophysiology of vWD? What Are the Subtypes?
			How Does Renal Failure Cause Coagulopathy?
			How Does Liver Disease Cause Coagulopathy?
			What Are the Vitamin K-Dependent Clotting Factors?
			How Is Thrombocytopenia Defined? How Do Various Platelet Count Thresholds Affect Bleeding?
			What Are the Causes for Thrombocytopenia?
			What Is the Mechanism of DIC?
			What Is Physiological Fibrinolysis?
			What Can Abnormal Activation (Pathological) of the Fibrinolytic Pathway Cause? How Is It Classified?
			What Coagulation Factors Do INR and PTT Measure, and What Drug Therapies Can They Monitor?
			Describe the Mechanism of the Commonly Used Anticoagulant Medications
		Workup
			Prior to an Elective Operation, What Is the Best Method to Identify a Patient at Risk for Bleeding During Surgery? Is Routine Laboratory Screening for Bleeding Disorders Necessary?
		Management
			What Is the First Step in the Management of a Patient with Suspected Postoperative Bleeding?
			At What Point Should Re-exploration Be Considered for a Patient Who Is Bleeding Postoperatively?
			How Is Bleeding Secondary to Renal Failure Corrected?
			How Is Bleeding Secondary to Liver Disease Corrected?
			At What Threshold Should Platelets Be Administered?
			What Is the Best Way to Urgently Reverse Warfarin?
		Complications
			What Are the Different Types of Transfusion Reactions, Their Causes, and Management?
			What Is the Leading Cause of Transfusion-Related Fatalities?
		Areas You Can Get in Trouble
			Failing to Stop Antiplatelets/Anticoagulants in a Timely Fashion Prior to Surgery
			Rebound Hypercoagulability
		Summary of Essentials
			History and Physical Examination
			Workup
			Management
			Complications
		Suggested Reading
	40: Postoperative Decreased Urine Output
		Diagnosis
			What Is the Differential Diagnosis for a Rise in Creatinine in the Postoperative Setting?
			What Is the Most Likely Cause for the Patient’s Decreased Urine Output?
		History and Physical Exam
			Why Is It Important to Review the Operative Record and the Anesthetic Record?
			What Is the Most Common Presentation of AKI?
			What Is the Difference Between Oliguria and Anuria?
			What Are the Most Common Nephrotoxic Medications?
		Physiology/Pathophysiology
			Which Patients Are at Greatest Risk for Intravenous Contrast-Induced AKI?
			What Is the Major Force Driving Filtration in the Kidney?
			Does Unilateral Ureteral Obstruction Lead to Renal Failure?
			Is It Common to Have Oliguria Following Major Surgery? If so, Why?
			Can Prerenal AKI Lead to Intrarenal AKI and Eventually Renal Failure?
			How Does General Anesthesia Affect Cardiac and Renal Function?
		Workup
			What Is the Best Initial Test When Suspecting AKI?
			What Other Tests Can Help Distinguish Between the Three Major Categories of AKI?
			How Do Specific Gravity and the Presence of Cells/Casts in a Urinalysis Help?
			What Are the RIFLE Criteria?
			What Imaging Is Useful in the Workup of Oliguria?
		Management
			When Encountering Low Urine Output in a Patient With an Indwelling Foley Catheter, What Needs to Be Ruled Out First as an Easily Correctable Cause?
			What Should Be Done Next?
			What Is a Fluid Challenge?
			What if the Patient Does Not Respond to Repeat Fluid Challenges? How Do You Assess Adequacy of Fluid Replacement?
			What Should Be Done if the Patient Is Suspected of Having a Postrenal Obstruction?
			What Are the Indications for Urgent/Emergent Dialysis?
			What Is the Role of Diuretics in the Treatment of Oliguria?
			What Is the Theoretical Role of Dopamine in the Treatment of Oliguria? What Is the Actual Role?
		Area You Can Get in Trouble
			MRI in Patient with Renal Failure
		Summary of Essentials
			History and Physical Exam
			Physiology/Pathophysiology
			Workup
			Management
		Suggested Reading
	41: Shortness of Breath 5 Days After Surgery
		Diagnosis
			What Is the Differential Diagnosis for Postoperative Shortness of Breath?
			What Is the Most Likely Diagnosis?
		History and Physical Exam
			What Is Virchow’s Triad? Which Part of the Triad Can Be Invoked in the Patient?
			What Are the Risk Factors for VTE and Their Mechanism?
			What Is the Wells Score for PE?
			What Are the Main Clinical Findings Associated with a DVT?
			What Is Homans’ Sign? Why Has It Fallen Out of Favor?
			What Are the Five Classic Causes (Five Ws) of Postoperative Fever and When in the Postoperative Setting Would Each Be Expected to Occur?
		Pathophysiology
			What Causes Atelectasis and How Is It Prevented?
			What Are the Most Commonly Inherited Causes of Hypercoagulability (Thrombophilia)?
			What Are the Most Common Acquired Causes of Hypercoagulability?
			What if a Patient Has No Obvious Component of Virchow’s Triad and Presents with Venous Thromboembolism (VTE)?
			What Is the Difference Between a Cardiac and Noncardiac Cause of Postoperative Pulmonary Edema?
			How Do You Distinguish Between Cardiogenic and Noncardiogenic Pulmonary Edema Based on History and Physical Exam? Why Is It Important to Distinguish Between Them?
			What Are the Three Routes by Which a Patient Develops Postoperative Pneumonia?
			What Is an A-a Gradient? What Is the Differential of a Wide A-a Gradient?
		Workup
			What Is the First Step in the Workup of a Patient Suspected of Having a Pulmonary Embolism?
			What Is the Full Diagnostic Algorithm for a Possible Pulmonary Embolism? (. Fig. 41.1)
			When There Is a High Suspicion of VTE, What Is the First Step in the Workup/Management?
			When Suspicion of VTE Is Low, What Is the First Step in the Workup?
			In a Patient with PE, What Are the Most Common Findings on ABG, ECG, and CXR?
			What Are the Classic, Though Uncommon, Findings on CXR and ECG That Are Associated with PE?
			Since CXR and ECG Are Often Normal with a PE, How Do They Help?
			What Are the Typical Findings on CT Angiography for PE? What Is the Sensitivity and Specificity?
			How Does V/Q (Ventilation/Perfusion) Scan Work? Why Has It Fallen Out of Favor for the Diagnosis of PE?
			If the Patient Is Critically Ill and Unable to Be Transported for Imaging, What Bedside Options Are There for the Indirect Diagnosis of PE?
			How Is the Severity of PE Classified?
		Management
			What Is the Initial Anticoagulant Management of PE?
			What if the Patient Has a History of or Suspected Heparin-Induced Thrombocytopenia (HIT)?
			What Are the Four Options for the Subsequent Treatment of PE?
			When Should tPA Be Considered?
			When Should Open Pulmonary Embolectomy Be Considered?
			What if the Patient Has a Contraindication to Anticoagulation?
			What Is the Recommendation for Long-Term Anticoagulation After First-Time VTE?
		Areas Where You Can Get in Trouble
			Anticoagulation in Patients with Malignancy
			Anticoagulation During Pregnancy?
			PE Can Mimic Sepsis
		Summary of Essentials
			History and Physical Exam
			Pathophysiology
			Differential Diagnosis
			Diagnosis
			Management
			Special Conditions
		Suggested Reading
XI: Question Set: Surgical
Complications
	Questions
	Answers
XII: Trauma
	42: Abdominal Pain Following Motor Vehicle Collision
		Diagnosis
			Is This Patient in Shock? What Are the Different Types of Shock?
		History and Physical Exam
			What Are the Clinical Manifestations of Hypovolemic Shock?
			What Is the Significance of Blood at the Urethral Meatus?
			What Is the Significance of Gross Hematuria?
		Pathophysiology
			How Much Blood Loss Is Necessary to Cause Hypotension (as in the Patient Above) in the Supine Position?
			What Are the Five Main Sources of Major Blood Loss in Blunt Trauma?
			What Are the Most Likely Injuries in Each of These Locations that Would Lead to Major Blood Loss?
			What Cavity Should Not Be Considered to Be the Source of Hemorrhagic Shock and Why?
		Initial Management
			What Are the ABCDE of Trauma Patient Management?
			What Is Included in the Secondary Survey of Trauma Patients?
			What Is the First Step in the Management of the Patient Presented?
			What Type of Airway Is Recommended in the Trauma Setting?
			What Are the Two Types of Surgical Airways? Which One Is More Appropriate in the Emergent Trauma Setting?
			Why Is Nasotracheal Intubation Not Recommended in the Trauma Setting?
			How Does One Confirm Proper Intubation?
			Where Should the Peripheral IV Lines Be Placed?
			Is a Central Line Preferred over a Peripheral IV for Trauma Patients?
			What Is the Ideal Fluid Resuscitation in Trauma?
			Why Are Large Doses of K + Not Given in the Initial Resuscitation?
			What Is Colloid? What Is the Theoretical Benefit? Any Actual Benefit in Trauma Resuscitation?
			After Appropriate Fluid Resuscitation Is Started, What Must Be Done Next?
			What Is the Role of Diagnostic Peritoneal Lavage (DPL) in the Trauma Patient?
		Subsequent Management
			What Is the Management of Intra-abdominal Bleeding Due to Splenic Injury?
			What Is the Management of Intra-abdominal Bleeding Due to Liver Injury?
			What Are the First Steps in the Management of Patients with a Pelvic Fracture? What Is the Subsequent Management?
		Areas You Can Get in Trouble
			Failing to Recognize the Significance of Free Fluid in the Peritoneal Cavity When There Is No Solid Organ Injury
			Treating Hypovolemic Shock Initially with Vasopressors
		Area of Controversy
			Management of a Pelvic Fracture in a Hemodynamically Unstable Patient
		Summary of Essentials
			Diagnosis
			History and Physical Exam
			Pathophysiology
			Initial Management
			Subsequent Management
			Watch Out
		Suggested Reading
	43: Penetrating Abdominal Trauma
		Diagnosis
			What Is the Most Likely Diagnosis?
		History and Physical
			What Is the MIVT Prehospital Report?
			What Are the Two Most Common Types of Penetrating Trauma?
			What Three Physical Exam Findings Independently Mandate Immediate Operative Intervention in Patients with Penetrating Abdominal Trauma?
			What Is a Tangential GSW? Do Patients with Findings of a Tangential GSW Require Any Further Workup?
		Anatomy
			What Are the Borders of the Anterior Abdomen, Flank, Back, and Thoracoabdomen? (. Table 43.1)
			What Is the Significance of Injuries to the Thoracoabdominal Region?
			What Three Distinct Regions Comprise the Internal Abdomen?
			Why Is It Difficult to Diagnose Injuries to Retroperitoneal Organs in Trauma Patients?
			What Is the Zone Classification for Retroperitoneal Hematomas and How Does the Location Affect Management?
			What Are the Two Most Common Organs Injured Following a Penetrating Abdominal Injury?
			What Injuries Must Be Ruled Out with GSWs That Travel Across the Pelvis?
		Workup
			What Initial Workup Is Recommended for the Above Patient?
			In the Absence of Shock, Peritonitis, or Evisceration, What Imaging Is Recommended for a Patient with Penetrating Abdominal Trauma?
			Are Diagnostic Peritoneal Lavage (DPL) and FAST Helpful Studies in Patients with Penetrating Abdominal Trauma?
			Is There a Role for Local Wound Exploration for Penetrating Abdominal Injury?
		Initial Management
			What Are the First Steps in the Management of This Patient?
			What Is a Massive Transfusion Protocol (MTP)? When Should It Be Initiated?
			What Is Permissive Hypotension?
			Should Empiric Antibiotics Be Routinely Administered to All Patients with Penetrating Trauma?
			Should Tetanus Prophylaxis Be Routinely Administered to Patients with Penetrating Trauma?
			How Do You Manage a Patient Who Presents with Impalement?
		Subsequent Management
			Why Are Trauma Patients Prepped and Draped From Chin to Knees in the Operating Room?
			What Is the Traditional Mainstay of Surgical Management for Patients with Penetrating Abdominal Trauma?
			Is There a Role for Laparoscopy Following Penetrating Abdominal Trauma?
			What Is the “Lethal Triad of Death” in Trauma?
			What Is Damage Control Surgery?
			When Is It Appropriate to Consider Nonoperative Management of Patients with Penetrating Abdominal Trauma?
			How Long Should Antibiotics Be Continued for Following a Trauma Laparotomy?
			Do Patients with Small Bowel or Colon Injuries Require a Stoma?
			What Is the Best Management for a Patient with an External Iliac Artery Injury in the Presence of a Contaminated Abdomen Due to a Colon Injury?
		Complications
			What Is Abdominal Compartment Syndrome? Who Is at Risk?
		Special Situations/Circumstances
			Thoracoabdominal Penetrating Wounds
		Summary of Essentials
			History and Physical Exam
			Anatomy
			Workup
			Initial Management
			Subsequent Management
			Complications
		Suggested Readings
	44: Pedestrian Struck by Motor Vehicle
		Diagnosis
			What Is the Most Likely Diagnosis?
		History and Physical Exam
			What Are “Hard” and “Soft” Signs of Vascular Injury?
			Do All Patients with Hard Signs Need to Go Emergently to the OR?
			What Are the 6 Ps of Acute Limb Ischemia?
			What Is the Implication of an Audible Bruit/Palpable Thrill Near an Artery in Association with Trauma?
			What Are the Principles of the Physical Examination of an Injured Extremity?
		Pathophysiology
			What Is the Mechanism of Popliteal Artery Injury?
			What Are the Classic Orthopedic Fractures/Dislocations that Are Associated with Arterial Injury? (. Table 44.2)
			Why Is It Important to Promptly Reduce a Dislocation?
			What Is a Minimal Vascular Injury?
			What Is Suggested by Numbness of the  Skin Overlying the Lateral Shoulder After Shoulder Dislocation?
		Workup
			Should the Knee Be Tested for  Ligamentous Injury?
			What Noninvasive Bedside Test Provides Objective Evidence of a Vascular Injury? How Is It Performed?
			If the ABI Is Abnormal, What Additional Vascular Imaging Study Is Recommended?
			What Additional Radiologic Imaging Study Is Recommended for a Posterior Knee Dislocation?
			In Penetrating Trauma, Should an Angiogram Be Performed If the  Bullet/Stab Wound Is Close to an  Artery, Even If There Are No Signs of Injury?
			What Is the Mangled Extremity Severity Score and How Is It Used?
		Management
			What Are the First Steps in the Management of This Patient?
			Is There a Role for Tourniquets in Patients with Life-Threatening Hemorrhage of an Extremity?
			What Is the Next Step in the Management of a Knee Dislocation?
			What Is the Next Step Following Reduction?
			What Are the Principles of Operative Management of a Popliteal Artery Injury in This Setting?
			If a Conduit Is Needed to Replace the Injured Artery, Where Is It Taken From?
			What If the Popliteal Vein Is Also Injured?
			What If There Is a Combined Orthopedic and Vascular Injury, Which Is Repaired First?
			Does Heparin Administration Help?
			What Is the Risk of Limb Loss with a Posterior Knee Dislocation and Popliteal Injury?
		Area Where You Can Get in Trouble
			Assuming There Is No Arterial Injury Because the Patient Has Palpable Pulses
		Area of Controversy
			The Role of Endovascular Repair
		Summary of Essentials
			History and Physical Exam
			Diagnosis
			Management
			Prognosis
		Suggested Readings
	45: Gunshot Wound to the Left Neck
		History and Physical Examination
			What Are the Most Important Parts of the History and Physical Examination for a Patient with a Neck Injury?
			What Are Hard Signs of Vascular Injury?
			What Are Other Nonvascular Hard Signs that May Warrant a Surgical Exploration?
			What Is the Significance of the Following Signs/Symptoms? (. Table 45.1)
			What Is the Diagnosis in This Patient?
		Anatomy
			What Are the Zones of the Neck and What Are Their Borders?
			What Key Anatomic Structures Are At Risk of Injury Within the Three Zones of the Neck? (. Table 45.2)
		Pathophysiology
			What Is the Significance of Whether or Not the Injury Has Penetrated the Platysma (Superficial Neck Muscle)?
			What Types of Arterial Injuries Do Bullet Wounds Cause?
			What Is a Pseudoaneurysm? How Does It Differ From a Hematoma?
			What Is an Arterial Intimal Injury? How Is It Managed? What Is a Dissection?
			What Nerve Would Be Injured if This Patient Presented with Vocal Cord Paralysis?
		Workup
			What Studies Are Recommended for Patients with Penetrating Neck Trauma?
		Management
			What Are the First Steps in the Management of a Penetrating Neck Wound?
			What Is the Next Step Following the Initial Management?
			What If There Are No Hard Signs of Injury?
			How Does the Zone of Injury Affect Management in Patients Without Hard Signs of Injury?
			What Are the Principles of Repair of a Carotid Injury?
			How Do You Manage an Injury to the Internal Jugular Vein?
			How Does the Type of Arterial Injury Seen on CTA Affect Management? (. Table 45.4)
		Areas Where You Can Get in Trouble
			With GSW, Bullets May Cross Zones As well as Sides of the Neck
			Not Addressing Airway First (Even with No Obvious Airway Compromise)
			Missing a Blunt Carotid Injury
		Summary of Essentials
			History and Physical Examination
			Anatomy
			Pathophysiology
			Workup
			Management
		Suggested Reading
	46: Stab Wound to the Chest
		Case Study
		Diagnosis
			What Are Considered the “Lethal” Six Injuries of Thoracic Trauma? (. Table 46.1)
			What Are Considered the “Hidden” Six Injuries of Thoracic Trauma? (. Table 46.2)
			How Is the Diagnosis of Tension Pneumothorax Established?
			How Is the Diagnosis of Cardiac Tamponade Established?
			What Is the Most Likely Diagnosis in This Patient?
		History and Physical
			What Is the Differential Diagnosis for a Combative Trauma Patient?
			What Is the Differential Diagnosis of the Absent Breath Sounds on the Left?
			What Is the Implication of a Penetrating Injury to the Chest that Is Above Versus Below the Nipple?
			Why Is It Important to Know the Type of Weapon Used in a Penetrating Injury?
			What Is the Concern Given that the Systolic and Diastolic Pressures in the Patient Presented Are So Close to Each Other?
			Why Is It Important to Roll the Patient Over?
			What Is the Significance of Air Bubbling from a Penetrating Chest Wound?
		What Is Subcutaneous Emphysema?
		Pathophysiology
			Why Is a Tension Pneumothorax Dangerous?
			What Is the Implication of Distended Jugular Veins?
			What Causes Hypotension in Cardiac Tamponade?
			What Is the Most Important Factor in the Development of Cardiac Tamponade?
			Does a Normal Abdominal Examination Rule Out an Intra-abdominal Injury? Why or Why Not?
			Why Are Penetrating Chest Trauma Patients at Increased Risk of Air Embolism? How Would It Present?
			Why Would a Trauma Patient Lose Pulses (Cardiac Arrest) After Induction or Intubation During General Anesthesia?
		Initial Management
			What Is the First Step in Management?
			What Is the Next Step?
			What Is a Needle Thoracostomy? What Is the Indication for Placement? Where Is It Placed? Why Is That Location Chosen? What Needs to Follow After It Is Placed?
			What Is a Tube Thoracostomy? What Is the Indication for Placement? Where Is It Placed?
			What Are the Goals and Guidelines for Resuscitative or Emergency Department Thoracotomy (EDT)?
			What Is the Next Step?
			Following the Primary Survey, What Is the Next Step in the Management?
			Once Cardiac Tamponade Is Highly Suspected, What Is the Next Step?
			Why Is Pericardiocentesis Not Recommended in the Trauma Setting?
			What Is the Role of a Subxiphoid Window?
			Are Vasopressors Recommended in the Management of Traumatic Cardiac Tamponade?
		Subsequent Management
			What Should Be Immediately Ordered in All Patients that Present with Combative Behavior?
			How Much Pleural Fluid Can the Diaphragm “Hide” in an Upright Chest Radiograph?
			What Is the Classic Description for Cardiac Tamponade on Chest Radiograph?
			How Does a Chest Tube Drainage System Work, and How Do You Look for a Leak?
			How Does Inspiring 100% O2 Help to Resolve a Pneumothorax?
			How Is a Sucking Chest Wound Managed?
			How Is Flail Chest Managed?
			What Is the Most Important Management in Patients with Rib Fractures?
			When Should Patients Undergo Rib Fixation?
			How Would You Manage a Traumatic Air Embolus?
			What Can Increase the Risk of Developing an Air Embolism in a Patient Arriving with Penetrating Chest Trauma?
			What Are the Classic Mechanisms and Imaging Findings for Aortic Injury?
		Complications
			What Is the Most Dangerous Complication Following Pericardiocentesis?
			What Nerve Is at Risk When Opening the Pericardium during EDT?
			How Is a Recurrent or Persistent Hemothorax Managed if Chest Tube Drainage Fails?
			What Should You Consider in Patients with a Pneumothorax that Continue to  Have a Large Air Leak Following Chest Tube Placement?
		Areas Where You Can Get in Trouble
			Long-Term Consequence of Missing a Diaphragm Injury
			Do All Patients with Pneumothorax Require a Chest Tube?
		Areas of Controversy
			How Accurate Is Focused Assessment with Sonography for Trauma (FAST) in the Setting of Penetrating Trauma?
			Should Prophylactic Antibiotics Be Given for Chest Tube Placement?
		Special Situations/Circumstances
			Following Chest Tube Placement for a Traumatic Pneumothorax/Hemothorax, When Is It Appropriate to Remove It? What Is the Main Risk During Removal?
			Initially Stable Patient with Multiple Rib Fractures Has Increasing Oxygen Requirements and Respiratory Distress
		Summary of Essentials
			History and Physical
			Differential Diagnosis
			Pathology/Pathophysiology
			Management
			Complications
		Suggested Reading
	47: Burns to the Face, Trunk, and Extremities
		Case Study
		Diagnosis
			What Is the Diagnosis and Resulting or Associated Complications Affecting This Patient?
		History and Physical
			What Are the Different Levels of Burn Injury and How Do They Present?
		What Are the Risk Factors for Burn Injuries?
		How Does One Determine the Severity of a Burn Injury?
			What Are the Criteria for Transferring the Patient to a Burn Center?
			What Is the Significance of Carbonaceous Sputum?
			What Is the Significance of Cherry-Red Skin in a Patient Rescued From a House Fire?
			Can a Second-Degree Burn Progress to a Third-Degree Burn?
			What Is the Significance of a Circumferential Burn in the Extremity? How About if on the Chest?
			What Population of Patients Has the Highest Morbidity From Burn Injuries?
			What Are the Greatest Risk Factors Associated with Increased Mortality in Burn Patients?
		Pathophysiology
			What Are the Different Causes of Burns?
			What Are the Physiologic Manifestations of a Burn in the First 24 h?
			Why Are Burn Patients at Increased Risk for Severe Volume Depletion?
			Why Are Burn Patients at Higher Risk for Gastrointestinal Ulcers?
			What Organisms Are Classically Involved in Burn Wound Infections?
		Workup
			What Is the First Step in the Evaluation of This Patient?
		How Is Inhalational Injury Definitively Diagnosed?
			What Is the Best Way to Evaluate for CO Poisoning?
			How Do You Diagnose a Burn Wound Infection?
		Management
			How Would You Manage a Patient with Inhalational Injury?
			How Do You Calculate the Appropriate Volume of Fluid Resuscitation for a Burn Victim in the First 24 h?
			What Is the Management for CO Poisoning?
			What Type of Fluid Should Be Used Acutely in a Burn Patient?
			What Electrolyte Abnormality Must Be Closely Monitored in Burn Patients?
			What Should Be Done for a Patient with a Circumferential Chest Burn and Deteriorating Respiratory Status?
			What Are the Other Indications for Escharotomy?
		How Do You Manage Burn Wounds?
			Should All Burn Patients Be Started on Prophylactic IV Antibiotics?
			What Topical Burn Agents Are Commonly Utilized in Burn Patients?
			What Medication Should All Burn Patients Be Started on to Prevent Curling’s Ulcers?
			Can Patients with Severe Burn Injuries Be Fed Orally? Why or Why Not?
			What Are the Principles of Management for Chemical Burns?
			What Is the Key Management Principle for Patients with Electrical Burns?
			Is There Benefit to Reducing the  Hypermetabolic State of Burn Patients?
		Areas You Can Get in Trouble
			Child Abuse
			Chronic Non-healing Wound
		Summary of Essentials
			History and Physical
			Pathophysiology
			Workup
			Management
		Suggested Reading
	48: Severe Right Leg Pain After Tibia Fracture
		Diagnosis
			What Is the Differential Diagnosis?
			What Is the Most Likely Diagnosis?
		History and Physical
			What Are the Six Ps of Compartment ­Syndrome? How Do These Ps Differ from Acute Limb Ischemia?
			What Are Considered Early Symptoms/Signs of Compartment Syndrome?
			What Is Meant by Pain Out of Proportion to the Physical Exam Findings (Clinical Situation)?
		Anatomy
			How Many Compartments Are There in the Upper Leg (Thigh)? Lower Leg? Upper Arm? Forearm?
		Pathophysiology
			How Are the Etiologies of Compartment Syndrome Classified?
			What Is the Pathophysiology of Compartment Syndrome?
			How Does Chronic Compartment Syndrome Differ from Acute Compartment Syndrome?
			What Is the Significance of the First Web Space Numbness?
			What Is the Implication of Pulselessness in the Setting of Compartment Syndrome?
			What Is Volkmann’s Ischemic Contracture?
			What Is the Pathophysiology of the Various Types of Compartment Syndrome?
			What Is Abdominal Compartment Syndrome? What Are the Risk Factors?
		Workup
			How Is the Diagnosis of Compartment Syndrome Established in an Extremity?
			What Is the Role of Measuring Compartment Pressures with Suspected Extremity Compartment Syndrome?
			How Is Abdominal Compartment Syndrome Diagnosed?
		Management
			What Is the Treatment of Extremity Compartment Syndrome?
			If Compartment Syndrome Is Suspected in Only One Compartment of the Lower Leg, Why Is Fasciotomy Performed for All Four Compartments?
			What Compartment in the Lower Leg Is Most Often Missed During Fasciotomy?
			What Is the Treatment of Abdominal Compartment Syndrome?
		Complications
			What Complications Can Occur from Lower Leg Fasciotomy?
		Summary of Essentials
			History and Physical
			Pathophysiology
			Workup
			Management
			Prognosis
		Suggested Reading
XII: Question Set: Trauma
	Questions
	Answers
XIII: Upper Gastrointestinal Bleeding
	49: Bloody Emesis
		Diagnosis
			What Is the Differential Diagnosis for an Upper GI Bleed?
			What Are the Most Likely Diagnoses in This Patient?
		History and Physical
			What Information Can the Color and Texture of Stool or Emesis Provide?
			What Variables Adversely Affect Prognosis in a Patient with an Upper GI Bleed?
		Pathophysiology
			What Are Esophageal Varices?
			What Are the Differences Between Acute and Chronic Gastritis?
			What Is Dieulafoy’s Lesion?
			What Are the Important Anatomic Correlations for Peptic Ulcer Disease?
		Workup
			Why Might the Hemoglobin/Hematocrit Be Normal in Spite of a Major GI Bleed?
			What Happens to the BUN/Creatinine Ratio During an Upper GI Bleed (UGIB)?
			What Part of the GI Tract Is Considered an UGIB?
			If a Patient Presents with Bloody Emesis and Bright Red Blood per Rectum, Is It an Upper or a Lower GI Bleed?
			What Is the Difference Between Obscure and Occult GI Bleeding?
			What Is a Common Cause of Obscure GI Bleeding and/or Anemia Unique to Patients with a Hiatal Hernia?
		Management
			What Are the First Steps in the Management of This Patient?
			What Is the Role of Placing an NG Tube?
			What Is the Difference Between a “Type and Screen” and a “Type and Cross”? Which One Is More Appropriate in This Setting?
			What if the Patient Needs Blood Immediately?
			How Should the Bed Be Positioned in a Patient Who Is Vomiting Blood?
			Following Resuscitation, What Is the Next Step in the Management?
			In What Setting Should the Patient with an UGI Bleed Be Managed?
			Following Stabilization, What Is the Next Step in the Management? What Is the  Optimal Timing?
			What if Upper Endoscopy Fails to Show the Source of a GI Bleed, What Other Imaging Might Be Useful?
			In the Absence of Varices, What Are the Therapeutic Endoscopic Options for Treating an UGI Bleed?
			What Are the Indications for Surgery in a Patient with an Upper GI Bleed?
			What Are the Surgical Options in a Patient with a Bleeding Ulcer that Fails Medical Management?
		Special Management Circumstances
			If an Ulcer or Gastritis Is Found on ­Endoscopy, What Additional Studies/­Therapies Are Recommended?
			How Do You Test for H. pylori Eradication?
			How Does the Management of UGI Bleed Differ for Esophageal Varices?
			What Is the Best Way to Prevent Recurrent UGI Bleed from Esophageal Varices?
			How Does the Management of UGI Bleeding Differ for a Mallory-Weiss Tear?
			What Is the Best Way to Prevent/Treat Stress-Related Mucosal Damage (Stress Ulcers and Stress Gastritis)?
			Why Would Patients Develop Nausea, ­Dizziness, and Flushing Postgastrectomy?
		Areas Where You Can Get in Trouble
			Assuming That UGI Bleeding in an Alcoholic Is Due to Esophageal Varices
			Sequelae of Ingesting Harmful Liquids
		Area of Controversy
			When Should Transfusion Be Given for an Acute UGIB?
		Summary of Essentials
			History and Physical Exam
			Pathology/Pathophysiology
			Differential Diagnosis
			Diagnosis
			Management
			Special Management Situations
			Prevention
		Suggested Reading
	50: Severe Epigastric Abdominal Pain
		Diagnosis
			What Is the Differential Diagnosis?
			What Is the Most Likely Diagnosis?
		History and Physical
			What Are the Most Common Symptoms in a Patient with Peptic Ulcer Disease (PUD)? How Might One Distinguish Gastric from Duodenal Ulcers?
			What Is the Typical Presentation for a Perforated Peptic Ulcer?
		Pathophysiology
			How Common Are Peptic Ulcer Perforations and How Often Do They Result in Death?
			What Is the Most Common Cause of Peptic Ulcers?
			How Do NSAIDs Lead to Peptic Ulcers?
			What Are Some Other Factors That May Lead to the Presence of Peptic Ulcers?
			What Are the Different Types of Peptic Ulcers and How Are They Categorized?
			How Does Vomiting Change a Patient’s Acid/Base Balance?
		Workup
			How Is the Diagnosis of Perforated PUD Established?
			What Laboratory Studies Should Be Sent?
			What Additional Diagnostic Studies Are Recommended for a Patient with  a Suspected Perforated PUD?
			What Is the Role of CT Scan?
			What Is the Role of an Upper GI Study (UGIS)?
			What Factors Increase Mortality in Perforated PUD?
		Management
			What Is the Basic Principle of Surgical ­Treatment for Perforated Peptic Ulcer?
			How Should a Perforated Peptic Ulcer Be Repaired?
			Is Conservative (Nonoperative) ­Management in Perforated PUD Ever Appropriate?
			What Are the Principles of Management of Nonoperative Management?
			Can Perforated Peptic Ulcers Be Treated Laparoscopically?
		Summary of Essentials
			History and Physical
			Pathophysiology
			Workup
			Management
		Suggested Reading
	51: Weight Loss and Early Satiety
		Diagnosis
			What Is the Differential Diagnosis for a Patient with Vague Abdominal Pain?
			What Is the Most Likely Diagnosis?
		History and Physical
			What Is the Prevalence of Gastric Cancer and What Causes It?
			Why Is the Mortality Rate so High for Gastric Cancer?
			What Are the Most Common Symptoms/Signs for a Patient with Gastric Cancer? What Are “ALARM” Symptoms/Signs?
			Are There Specific Findings on Physical Exam for Gastric Cancer?
		Pathophysiology
			What Is the Vascular Supply to the Stomach?
			Why Do Patients with Gastric Cancer Get Iron Deficiency Anemia?
			What Is an Intestinal-Type Gastric Adenocarcinoma?
			What Is a Diffuse-Type Gastric Adenocarcinoma?
			What Is Linitis Plastica?
			What Is a Gastrointestinal Stromal Tumor (GIST)?
			Does Lymphoma Occur in the Stomach?
		Workup
			What Is the Best Way to Work Up a Patient with Suspected Gastric Cancer?
			Once the Diagnosis of Gastric Cancer Is Established, What Further Staging Is Recommended?
		Management
			How Is Gastric Cancer Treated?
			What Is the Role of HER-2 Gene Amplification and Chemotherapy?
		Complications
			What Are the Common Complications that Can Occur After a Gastric Resection?
			How Are Anastomotic Leaks Identified and Treated?
		Summary of Essentials
			History and Physical
			Pathophysiology
			Workup
			Management
			Prognosis
		Suggested Reading
	52: Chest Pain After Vomiting
		Diagnosis
			What Is the Most Likely Diagnosis?
			What Is the Most Likely Diagnosis?
		History and Physical
			What Are the Risk Factors for Boerhaave’s Syndrome?
			Why Is Boerhaave’s Syndrome so Often Unrecognized?
			What Is Mackler’s Triad?
			What Is the Most Specific Sign of an Esophageal Rupture?
			How Does Boerhaave’s Syndrome Differ from Mallory-Weiss?
		Pathophysiology
			What Are the Most Common Causes of Esophageal Perforation?
			Why Do Patients with Boerhaave’s Syndrome Become So Septic?
		Workup
			What Is the First Step in Workup for a Patient Suspected of Having Boerhaave’s Syndrome?
			What Is the Role of Oral Contrast Studies in Diagnosing Boerhaave’s Syndrome? How About CT?
			What Type of Contrast Should Be Used?
			Is There a Role for Endoscopy During Diagnosis?
		Management
			What Are the Initial Steps in the  Management of a Patient with Boerhaave’s Syndrome?
			What Is the Sepsis Bundle?
				Surviving Sepsis Bundle
			Does the Time Interval Between Perforation and Intervention Matter?
			What Constitutes Conservative Management, and Which Patients Are Candidates for It?
			What Are the Surgical Options for a Patient with Boerhaave’s Syndrome?
			What Is the Prognosis of a Patient with Boerhaave’s Syndrome?
		Area of Controversy
			What Are the Endoscopic Options for a Patient with Boerhaave’s Syndrome?
		Summary of Essentials
			History and Physical
			Pathophysiology
			Workup
			Management
		Suggested Reading
XIII: Question Set: Upper
Gastrointestinal
	Questions
	Answers
XIV: Urology
	53: Scrotal Pain
		Diagnosis
			What Is the Differential Diagnosis?
			What Is the Most Likely Diagnosis for this Patient?
		History and Physical
			What Is the Cremasteric Reflex?
			What Would Cause an Absent Cremasteric Reflex?
			What Is Prehn’s Sign? Is It Reliable?
			What Is the Blue-Dot Sign?
			What Are the Four Cardinal Symptoms and Signs of Testicular Torsion?
			What Are the Important Differences Between Testicular Torsion and Appendix Testes Torsion?
			Does a History of Trauma or Presence of Fever Rule Out Testicular Torsion?
		Pathology/Pathophysiology
			What Congenital Defect Predisposes Children to Developing Testicular Torsion?
			What Are Other Risk Factors for Testicular Torsion?
			How Does One Distinguish Testicular Torsion from Acute Epididymitis?
		Workup
			If Suspicion for Testicular Torsion Is High, What Laboratory Tests Are Important to Obtain? What Is the Role of Additional Imaging?
			If Suspicion of Testicular Torsion Is Low, What Laboratory Tests Should Be Obtained? What Imaging Is Useful?
			In the Trauma Setting, What Are the Most Important Things to Look for on Doppler Ultrasound?
		Management
			In the Setting of Suspected Testicular Torsion, What Is the Optimal Timing from Initial Evaluation to Definitive Management?
			Is There a Role for Manual Detorsion in the Emergency Department?
			What Is the Surgical Treatment for Testicular Torsion?
			What if the Testicle Is Necrotic at the Time of Surgical Exploration?
			What Is the Recommended Management for Minor Trauma to the Testes?
			What Are the Operative Indications for Testicular Trauma?
		Prognosis
			Does Loss of a Testicle from Torsion Affect Fertility?
		Areas Where You Can Get in Trouble
			Failing to Perform a Scrotal Exam in an Adolescent Male with Abdominal Pain and Vomiting
			Penile Pain After Sexual Intercourse
		Summary of Essentials
			Diagnosis
			History and Physical
			Pathophysiology
			Workup
			Management
		Suggested Reading
	54: Scrotal Mass
		Diagnosis
			Why Is the Location of a Scrotal Mass Important to Identify?
			Describe the Cause(s) of Scrotal Masses Found Involving the Skin
			What Is the Differential Diagnosis of Scrotal Masses Involving the Spermatic Cord?
			What Is the Differential Diagnosis of Scrotal Masses Involving the Epididymis?
			What Is the Differential Diagnosis of Scrotal Masses Found Involving the Testes?
			What Is the Most Likely Diagnosis for This Patient?
		History and Physical
			What Features on History and Physical Examination Favor the Diagnosis of Testicular Cancer?
			What Is the Implication of Constitutional Symptoms in Association with a Painless Testicular Mass?
			What Risk Factors for Testicular Cancer Should Be Obtained on History?
			What Features on Physical Examination Favor a Nonmalignant Etiology?
			What Are the Main Diagnoses to Consider in the Presence of a Very Painful Scrotal Mass?
			What Benign Processes Are Typically Painless?
			What Physical Exam Maneuver Can Help Identify a Varicocele?
		Pathology/Pathophysiology
			Does Cryptorchidism Increase the Risk of Developing Testicular Cancer in the Undescended Testicle, the Contralateral Descended Testicle, or Both?
			What Is the Most Common Subtype of Testicular Tumor? Is It Malignant?
			What Are the Major Pathologic Subtypes of Testicular Cancers?
			What Causes Gynecomastia in Patients with Testicular Cancers?
			What Are the Differences Between Epididymitis and a Spermatocele?
			Why Does a Varicocele Form? Why Is It More Often on the Left? Why Does It Affect Fertility?
			What Is the Pathophysiology of a Hydrocele?
		Workup
			What Is the Key Imaging Modality for a Patient with a Painless Testicular Mass?
			Once a Solid Testicular Mass Is Identified, What Additional Imaging Is Recommended?
			What Relevant Blood Tests Should Be Obtained in a Patient with Testicular Cancer?
			Is Percutaneous Biopsy for a Testicular Tumor Recommended?
			What Are the Key Differences Between a Seminoma and Nonseminoma?
		Management
			How Is Pathologic Confirmation of Testicular Cancer Determined?
			Why Is an Inguinal Incision Preferred Over a Scrotal One?
			What Other Treatment Modalities Are Utilized After Initial Surgery for Testicular Cancer?
			What Should Be Recommended for Patients About to Undergo Orchiectomy and/or Chemotherapy?
			Does Unilateral Orchiectomy Lead to Impotence?
		Summary of Essentials
			History and Physical
			Pathology/Pathophysiology
			Workup
			Management
		Suggested Reading
	55: Blood in Urine
		Diagnosis
			What Is the Differential Diagnosis for Gross Hematuria?
			What Are the Most Likely Diagnoses for This Patient?
		History and Physical
			What Are the Differences Between Gross and Microscopic Hematuria?
			Why Is the Color and Consistency of the Urine Important?
			Other than Blood, What Can Make Urine Appear Red?
			Does It Matter if Hematuria Is at the Beginning of the Urine Steam or at the End?
			What Is the Importance of Pain in Association with Hematuria?
			What Is the Classic Presentation for Nephrolithiasis?
			What Are the Risk Factors for Nephrolithiasis?
			What Are the Most Common Symptoms/Presentation for Kidney Cancer? What Is the Classic Presentation?
			What Are the Main Risk Factors for Renal Cancer?
			What Is the Most Common Presentation for Bladder Cancer? What Are the Risk Factors?
			What Is the Most Common Presentation for Prostate Cancer? What Are the Risk Factors?
		Anatomy
			What Comprises the Urinary Tract and Where Along the Tract Can Bleeding Arise?
		Pathology/Pathophysiology
			What Is the Difference Between Glomerular and Non-glomerular Hematuria? Why Is It Important to Distinguish Them?
			Kidney Stones
				Where Do Renal Stones Develop and in What Circumstances Do They Lead to Symptoms?
				Why Do High Protein Diets Increase the Risk of Developing Renal Stones?
				What Is the Most Common Type of Renal Stone and What Is the Leading Cause of This Type of Stone?
			Renal Cancer
				What Is the Most Common Type of Kidney Cancer?
				Where Is the Most Common Location for RCC Metastasis?
				What Genetic Syndromes Are Associated with RCC?
				What Paraneoplastic Syndromes Are Associated with RCC?
			Bladder Cancer
				What Is the Most Common Type of Bladder Cancer?
			Prostate Cancer
				What Is the Most Common Type of Prostate Cancer?
		Workup
			What Is the First Step in the Workup of a Patient with Gross Hematuria?
			What Is the Next Step in the Workup?
			What Additional Laboratory Tests Should Be Ordered During the Hematuria Workup?
			How Does the Urinalysis Help Distinguish Between a Glomerular and a Non-glomerular Cause of Hematuria?
			What Workup Is Recommended for Non-glomerular Hematuria?
			What Workup Is Recommended for Symptomatic Non-glomerular Microscopic Hematuria?
			What Is the Best Diagnostic Test for Nephrolithiasis?
			What Further Workup Is Recommended in the Presence of Gross Hematuria?
			Does Negative Urine Cytology Rule Out a Malignancy?
		Management
			What Is the Guiding Treatment Principle for Hematuria?
			Renal Stones
				What Is the Best Management for Nephrolithiasis? How Does the Size of the Stone Change Management?
				What Are the Emergent Surgical Indications for Renal Stones?
			Renal Masses
				What Are the Treatment Options for Renal Masses Concerning for RCC?
				What Is a Radical Nephrectomy?
			Bladder Cancer
				How Is Urothelial Cell Carcinoma of the Bladder Treated?
				What Is a Radical Cystectomy?
			Prostate Cancer
				What Is a Radical Prostatectomy? Which Patients Are Appropriate Candidates for This Procedure?
				What Are Nonsurgical Treatments for Prostate Cancer?
		Special Circumstances
			What Are the Next Steps in Management if Significant Hematuria Persists Despite Foley Catheter Placement and Manual Irrigation?
			When Should a Patient Be Admitted to the Hospital for Gross Hematuria?
		Areas You Can Get into Trouble
			Not Placing a Foley Catheter in the Setting of Significant Gross Hematuria
			Placing a Foley Catheter in the Setting of Trauma and Blood at the Urethral Meatus
			Can Cross Hematuria Be Associated with an Abdominal Aortic Aneurysm (AAA)?
		Summary of Essentials
			History and Physical
			Pathology/Pathophysiology
			Workup
			Management
		Suggested Reading
XIV: Question Set: Urology
	Questions
	Answers
XV: Vascular
	56: Transient Loss of Vision in the Right Eye
		Diagnosis
			What Is the Differential Diagnosis for Transient Monocular Vision Loss?
			What Is the Most Likely Diagnosis for This Patient?
		History and Physical Examination
			What Is the Term for the Eye Symptom the Patient Describes?
			In a Patient with This Eye Symptom, What Finding on Fundoscopic Ophthalmologic Examination Would Support a Carotid Artery Source of the Symptom?
			What if the Patient Instead Described Transient Motor and Sensory Loss of the Left Arm and Leg, What Term Would Be Used for Such a Symptom?
			How Does One Distinguish a TIA from a Stroke?
			What Does a Carotid Bruit Signify?
			What Else Can Cause a Bruit in the Neck?
			What Is Subclavian Steal Syndrome?
			Are Dizziness, Syncope, and Headaches Considered Symptoms of Carotid Stenosis?
			What Are the Risk Factors for Carotid Stenosis?
		Anatomy
			How Do You Distinguish Between the ICA and External Carotid Artery (ECA) in the Neck?
			What Are the Branches of the ECA?
			What Is the First Branch of the Internal Carotid Artery?
			What Is the Classic Anterior Circulation Ischemic Stroke in Terms of Arterial Distribution and Symptoms?
			Why Is It Important to Distinguish Between Anterior and Posterior Circulation Symptoms?
		Pathophysiology
			What Are the Two Main Causes of Stroke?
			What Are the Main Sources of Cerebral Emboli?
			What Is a Lacunar Stroke?
			How Is an Atherosclerotic Plaque Formed?
			Why Does Atherosclerotic Plaque Localize at the Bifurcation of the Carotid Artery?
			How Does a Carotid Stenosis Typically Cause Symptoms?
			Is the Degree of ICA Stenosis Related to the Risk of Symptoms Developing?
			If a Patient Had Multiple TIAs, with the Same Symptom Each Time, What Is the Most Likely Source of the TIA?
			What Happens When the Internal Carotid Artery Occludes?
			What Is a Symptomatic Carotid Artery Stenosis?
			What Additional Symptom Can Help Differentiate Left from Right Carotid Artery Stenosis?
		Workup
			How Useful Is the Presence of a Bruit for Detecting Carotid Artery Stenosis?
			What Is the Diagnostic Test of Choice for the Workup of Possible Carotid Stenosis?
			Are Additional Diagnostic Studies Warranted?
		Management
			What Are the Three Management Options for Symptomatic Carotid Stenosis?
			What Are the Determinants of Which Option to Choose?
			At What Percent of ICA Stenosis Should CEA Be Considered in Symptomatic Patients?
			Following a Stroke or TIA, What Is the Optimal Timing of CEA?
			Why Is CEA Not Recommended for a Symptomatic Patient with an Occluded (100% Stenosis) ICA?
			What Is the Role of CEA in a Patient with Symptomatic ICA Stenosis that Resulted in a Stroke that Has Caused Complete Paralysis of the Arm and Leg As Well As Aphasia?
			What Is the Anticipated Stroke Risk Reduction for CEA for a Symptomatic 70–99% ICA Stenosis?
			What Is the Management of Carotid Stenosis in an Asymptomatic Patient?
			At What Percent of ICA Stenosis Should CEA Be Considered in Asymptomatic Patients?
			What Is the Anticipated Stroke Risk Reduction for CEA for an Asymptomatic 60–99% ICA Stenosis?
			Why Is There Less Benefit from CEA in Women?
			What Three Drugs Have Been Shown to Reduce the Risk of Perioperative Stroke After CEA?
			Why Is Blood Pressure Control Important Prior to CEA?
			What Complication Likely Occurred in a Patient Complaining of a Severe Headache Days After CEA?
			What Cranial Nerves Are at Risk of Injury During CEA?
			What Is an Acceptable Risk of Stroke/Death Following CEA?
		Area Where You Can Get in Trouble
			New Neurologic Deficit in the Postoperative Recovery Room After CEA
		Area of Controversy
			Carotid Stenting Versus CEA for Symptomatic High-Grade (>70%) ICA Stenosis
		Summary of Essentials
			Differential Diagnosis
			History
			Physical Exam
			Pathology/Pathophysiology
			Workup
			Management
		Suggested Reading
	57: Right Calf Pain with Walking
		Diagnosis
			What Is in the Differential Diagnosis?
			What Is the Most Likely Diagnosis?
		History and Physical
			What Is Claudication, and What Is the Three-Part Definition that Should Be Obtained by History?
			Claudication Is a Symptom of What Underlying Disease?
			What Are the Main Risk Factors for PAD?
			What Is the Spectrum of Severity in PAD?
			What Is Ischemic Rest Pain, and How Does It Present?
			What Is Buerger’s Sign?
			What Is the Differential Diagnosis of Ischemic Rest Pain?
			How Many Pulses Should Be Examined?
			In Addition to a Pulse Deficit, What Other Findings on Leg Examination Would Support PAD?
		Anatomy
			What Muscle Groups Are Affected By Claudication?
			Where Is the Hunter/Adductor Canal Located?
		Pathophysiology
			What Is Dependent Rubor?
			Why Is the Onset of Pain Consistently at the Same Walking Distance?
			What Other Disease Processes Can Cause Claudication?
			What Condition Is Classically Associated with Claudication in the Upper Extremities?
			What Is Buerger’s Disease?
			What Is Leriche’s Syndrome (Aortoiliac Occlusive Disease)?
		Workup
			What Is the Next Step in the Diagnostic Workup?
			How Is the ABI Measured?
			How Is PAD Defined?
			What Is the Typical ABI of a Patient with Claudication? Rest Pain?
			What Additional Studies Are Recommended?
			Is There Any Value in Screening an At-Risk Population for Asymptomatic PAD?
		Prognosis
			What Is the Risk of Limb Loss for Patients with Claudication?
			What Is the Risk of Limb Loss for Patients with Ischemic Rest Pain in the Foot?
			What Symptoms/Signs Are Considered Limb-Threatening PAD and Thus an Indication for More Aggressive Management?
		Management
			What Is the Initial Management of Claudication?
			Are There Any Drugs Specifically Approved by the Food and Drug Administration (FDA) for Claudication?
			What About a Statin? Does It Improve Claudication?
			What About Aspirin? Does It Improve Claudication?
			What About Clopidogrel?
			What About Anticoagulants Such As Heparin or Warfarin?
			What Are the Invasive Therapeutic Options?
			If Medical Management Fails, Which Patients with Claudication Are Candidates for an Invasive Therapeutic Approach?
			Which Patients with PAD Should Go Directly to an Invasive Therapeutic Approach Rather than an Initial Attempt at Medical Management?
			What Is the 5-Year Mortality for Patients with PAD?
		Key Areas Where You Can Get in Trouble
			Confusing the Dependent Rubor of Rest Pain with Cellulitis (or Confusing Dependent Rubor with a Well-Perfused Foot)
			Assuming that an ABI of 1.0 Is a Sign of Normal Circulation in a Diabetic Patient
		Areas of Controversy
			Should a Patient with Claudication Who Is Still Smoking Undergo an Invasive Procedure?
			In a Patient with Advanced Chronic Limb Ischemia (Tissue Loss, Rutherford Class 5 or 6), Is It Better to Attempt Endovascular Approaches (Angioplasty/Stenting) or to Perform an Open Surgical Bypass (Using Reverse Saphenous Vein)?
		Summary of Essentials
			History
			Physical Exam
			Pathology/Pathophysiology
			Differential Diagnosis
			Diagnosis
			Management
			Prognosis
			Special Situations
			Watch Out
		Suggested Reading
	58: Sudden Onset of Severe Left-Sided Abdominal Pain
		Case Study
		Diagnosis
			What Is the Differential Diagnosis?
			What Is the Most Likely Diagnosis?
		History and Physical Examination
			What Is the Typical Presentation for an Unruptured AAA?
			What Are the Risk Factors for AAA?
			Is Diabetes a Risk Factor for AAA?
			Is There a Role for Physical Examination in the Detection of AAA? What Are the Limitations?
			Is There a Role for AAA Screening? If So, Who Should Be Screened and How Often?
			In a Patient with an AAA, What Other Arteries Might Have Aneurysms? How Would You Screen for Them? What Is the Main Risk Associated with Those Aneurysms?
		Pathophysiology
			What Is the Normal Diameter of the Infrarenal Aorta in Men? Women?
			What Is the Primary Defect in AAA?
			At What Diameter Is the Infrarenal Aorta Considered to Be Aneurysmal?
			What Is the Average Annual Growth Rate of AAA?
			What Factors Influence Growth Rate?
			What Are Matrix Metalloproteinases (MMPs), and What Is Their Role in AAA Formation?
			Is There a Genetic Component to AAA Formation?
			If an AAA Ruptures, Where Does It Typically Do So?
			What Is the Relationship Between AAA Rupture and Size?
			Other Than AAA Size, What Are Other Risk Factors for AAA Rupture?
			What Are the Primary Differences Between an Aortic Dissection and an AAA?
		Workup
			What Is the First Step in the Evaluation of a Patient with a Suspected Ruptured AAA?
			In a Patient with Suspected Ruptured AAA, What Is the Goal of Fluid Resuscitation?
			What Imaging Is Recommended for Suspected Ruptured AAA in a Hemodynamically Stable Patient?
			How Does the Imaging of Choice Differ in a Hemodynamically Unstable Patient?
		Management
			How Does AAA Size and Growth Rate Influence Management for an Asymptomatic AAA?
			Non-ruptured Asymptomatic AAA
				Why Wait for the AAA to Reach this Size, Why Not Repair a Smaller AAA Since the Patient Is Theoretically Healthier and Is Younger?
				What Surgical Options Are Available for Asymptomatic AAA?
				How Do the Two Surgical Approaches Compare in Terms of Perioperative Morbidity and Mortality? What About Long-Term Mortality?
			Ruptured AAA
				What Are the Surgical Options for a Patient with a Ruptured AAA?
				What Makes an AAA Unsuitable for EVAR?
				What Is the Mortality Risk if an AAA Ruptures?
			What Is the Main Achilles Heel of Endovascular Repair?
			How Should Patients Be Monitored After an Endovascular AAA Repair?
		Areas Where You Can Get in Trouble
			Assuming that an AAA Is Not Ruptured Because No Contrast Extravasation Is Seen on CT
			Ignoring Early-Onset Diarrhea After AAA Repair
			Not Considering Aortoenteric Fistula as a Cause of GI Bleed After AAA Repair
			Ipsilateral Flank or Back Pain Following EVAR
		Area of Controversy
			Since EVAR Has a Lower Risk of Perioperative Mortality than Open Repair, Should We Lower Our Threshold for AAA Repair (i.e., < 5.5 cm) if the Patient Is an EVAR Candidate?
		Summary of Essentials
			History and Physical Examination
			Diagnosis
			Screening
			Management
			Complications
		Suggested Reading
	59: Cold, Painful Right Lower Extremity
		Case Study
		Diagnosis
			What Is the Differential Diagnosis?
			What Is the Most Likely Diagnosis?
		History and Physical
			What Is the Difference in Timing Between Acute and Chronic Limb Ischemia?
			What Are the Six “Ps” of Acute Limb Ischemia?
			Why Is the Cardiac History and Physical Important in Determining the Etiology of ALI?
			What Are the Key Findings that Point Toward an Embolic Etiology Rather than a Thrombotic Event?
			Why Is It Important to Examine the Contralateral, Nonischemic Limb?
			How Would You Best Assess This Patient’s Atherosclerotic Disease History?
			Why Is It Important to Ask About Past Interventions for PAD?
		Pathophysiology
			Where in the Arterial Tree Do Cardiac Emboli Tend to Lodge?
			How Do Aneurysms Cause Acute Limb Ischemia?
			How Does Peripheral Arterial Disease Lead to Acute Limb Ischemia?
			How Long Can Muscle Tissue Withstand Ischemia Before Irreversible Damage Occurs?
		Workup
			What Does a Doppler Probe Detect and What Are the Three Signals?
			Is the Ankle-Brachial Index Useful for ALI?
			How Is the Severity of Acute Ischemia Determined?
			What Imaging Is Recommended?
			What Additional Imaging Should Be Done if an Embolus Is Suspected as the Cause for ALI? If a Paradoxical Embolus Is Suspected?
		Management
			What Are the Three Most Important Initial Management Steps?
			What Are the Main Therapeutic Options for Acute Limb Ischemia?
			What Are the Absolute Contraindications to Thrombolytic Therapy? Relative Contraindications?
			How Does One Choose Between Open Surgery and Endovascular Therapy?
			Who Is the Best Candidate for Surgical Balloon Embolectomy?
			What Is the Treatment for Irreversible Limb Ischemia?
			What Are the Potential Dangerous Sequelae of Reperfusion Following ALI?
		Areas Where You Can Get into Trouble
			Mistaking the Diagnosis of ALI as a Neurological Condition
			Failing to Monitor for Elevated CPK and Myoglobinuria
			Following Revascularization, the Patient Develops Peaked T Waves on the EKG
			Following Revascularization, the Patient Develops a Swollen Leg
			Platelet Count Drop 5 Days After Successful Reperfusion
		Summary of Essentials
			History and Physical
			Pathophysiology
			Diagnosis
			Management
			Complications
			Areas Where You Can Get into Trouble
			Areas of Controversy
		Suggested Reading
XV: Question Set: Vascular
	Questions
	Answers
Index




نظرات کاربران